You are on page 1of 43

Nephrology - Item 1

A 20-year-old woman is evaluated during the third trimester of an otherwise uneventful first
pregnancy because of right calf tenderness. Medical history is unremarkable. On physical
examination she is anxious afebrile and slightly tachypneic. !er respiratory rate is "#$min and
blood pressure is "0%$&% mm !g. !er abdomen is slightly tender. !er right calf is tender but no
venous cord is pal-pable. !oman's sign is negative.
(aboratory studies)
!ematocrit *"+
(eukocyte count ,000$-(. normal differential
/latelet count 220000$-(
0lood urea nitrogen % mg$d(
1erum creatinine 0.# mg$d(
1erum sodium "*2 me2$(
1erum potassium *., me2$(
1erum chloride ,% me2$(
1erum bicarbonate 20 me2$(
Arterial oxygen saturation ,3+
4rinalysis p! #.0 specific gravity ".020. dipstick negative. normal microscopic
examination
5hich of the following clinical scenarios is most likely based on this patient's serum electrolyte
values6
7A8 Acute respiratory alkalosis secondary to a possible pulmonary embolism
708 Metabolic acidosis due to a gastrointestinal disorder
798 !yperchloremic metabolic acidosis caused by renal tubular acidosis
7:8 ;ormal electrolyte values in a pregnant woman
Nephrology - Item 2
A #3-year-old man has a "0-year history of hypertension hypercholesterolemia treated with
lovastatin and intermittent claudication diagnosed 2 years ago. :espite treatment with b-blockers
and diuretics his blood pressure readings are typically "30$"00-"0% mm !g and have never
reached the goal of "<0$,0 mm !g when measured in his physician's office. !is serum creatinine
concentration is ".# mg$d(. =nalapril "0 mg daily is started and his blood pressure falls to
"#0$"00 mm !g. >he dose of enalapril is subse2uently increased to 20 mg daily. >he patient is seen
2 weeks later. On physical examination his pulse rate is ,3$min and regular and blood pressure is
"*0$&0 mm !g without orthostatic changes. >here are no signs of volume overload. An epigastric
bruit is heard on abdominal examination. !is abdomen is nontender and there are no enlarged
organs. /opliteal pulses are intact but the pedal pulses are decreased. >here is no discoloration of
the toes or pedal edema.
(aboratory studies)
0lood urea nitrogen 30 mg$d(
1erum creatinine *.2 mg$d(
1erum sodium "*% me2$(
1erum potassium %.2 me2$(
1erum chloride ,, me2$(
1erum bicarbonate 20 me2$(
4rinalysis 1pecific gravity ".0"#. trace protein no blood ketones or glucose. normal
microscopic examination
5hich of the following is the most appropriate first step in managing this patient's acute renal
failure6
7A8 :iscontinue enalapril. measure serum potassium and creatinine levels
708 :iscontinue the b-blocker. measure serum potassium and creatinine levels
798 :iscontinue lovastatin. measure the serum creatinine level
7:8 Administer intravenous normal saline
7=8 Obtain emergent renal arteriography
Nephrology - Item 3
A &&-year-old woman is evaluated because of dysuria urinary fre2uency pyuria and fever. On
physical examination the patient weighs %< kg 7"", lb8. !er temperature is *3.0 ?9 7"00.<
?@8. >he remainder of her physical examination is normal. !er baseline serum creatinine value
is ".2 mg$d(. 4rine culture grows greater than "00000 colo-nies of =scherichia coli. Aou
diagnose cystitis and prescribe trimethoprim-sulfamethoxaBole orally twice daily for & days. On
day < of treatment the patient feels well has no fever or urinary symptoms and has no
erythrocytes or leukocytes in her urine. !owever her serum creatinine value is now ".3
mg$d(.
5hich of the following statements is correct regarding her baseline serum creatinine value6
7A8 >he serum creatinine value of ".2 mg$d( represents a normal glomerular filtration rate
7C@D8
708 >he serum creatinine value of ".2 mg$d( represents a C@D of less than %0+ of normal
798 0ecause of changes in the catabolism of creatinine in older persons the serum creatinine
value cannot be used to estimate the C@D in this patient
7:8 0ecause of the effects of aging on renal function this patient has an abnormally low C@D
regardless of her serum creatinine value
Nephrology - Item 4
@or the patient in the preceding 2uestion which of the following is the most likely cause of the
increase in her serum creatinine value from ".2 mg$d( to ".3 mg$d(6
7A8 Acute interstitial nephritis due to trimethoprimsulfamethoxaBole
708 Acute renal failure due to crystalliBation of sulfonamides in the renal tubules
798 0ladder outlet obstruction
7:8 Deduced tubular secretion of creatinine due to trimethoprim
Nephrology - Item 5
A <#-year-old white man comes to the emergency department because of cough and
shortness of breath of several hours' duration. >he patient who smokes cigarettes has had
bilateral ankle and leg edema for 2 months. !e has no history of heart disease.
On physical examination his temperature is *#., ?9 7,3.< ?@8. !is pulse rate is "02$min
respiratory rate is 2#$min and blood pressure is "%0$32 mm !g. !e has periorbital edema.
>here is no neck vein distention. 9ardiac examination discloses a normal 1"and 12 without
murmurs or gallops. >here are decreased breath sounds at the lung bases but no crackles are
heard. !is abdomen is soft and nontender. >here is <E edema of ankles and pretibial edema.
(aboratory studies)
!ematocrit <&+
(eukocyte count ,000$-(
/latelet count *2%000$-(
/lasma glucose 33 mg$d(
0lood urea nitrogen 2< mg$d(
1erum creatinine ".2 mg$d(
1erum sodium "*# me2$(
1erum potassium *.3 me2$(
1erum chloride "0< me2$(
1erum bicarbonate 2# me2$(
/lasma cholesterol #*% mg$d(
1erum albumin ".# g$d(
Arterial blood gas studies 7patient breathing room air8)
p! &.<3
/co2 23 mm !g
/o2 3< mm !g
0icarbonate 2# me2$(
4rinalysis <E protein several erythrocytes$hpf
A chest radiograph shows basal atelectasis. An electrocardio-gram reveals sinus tachycardia
with no other abnormalities. A ventilation-perfusion lung scan is reported as indeterminate for
pulmonary embolism.
5hich of the following diagnostic studies should be done next6
7A8 =chocardiography
708 :oppler ultrasonography of the kidneys and renal vasculature
798 Measurement of serum total complement and complement 9* concentrations
7:8 /ulmonary function tests
Nephrology - Item 6
A %*-year-old man with a long history of chronic alcohol abuse comes to the emergency
department because of a *-week history of increasing weakness anorexia and a productive
cough. On physical examination the patient is thin and is obviously dyspneic. !is pulse rate is
30$min and regular. 0lood pressure is "*0$&0 mm !g supine falling to "20$#% mm !g when
he stands. >here is a grade 2$# systolic murmur and an 1* gallop. 9rackles are heard over the
right hemithorax. >race peripheral edema is noted.
Fnitial laboratory studies)
0lood urea nitrogen "2 mg$d(
1erum creatinine ".0 mg$d(
1erum sodium "*# me2$(
1erum potassium *.# me2$(
1erum chloride "00 me2$(
1erum bicarbonate 22 me2$(
A chest radiograph shows an enlarged heart and a right middle lobe infiltrate or mass. >he
patient is hospitaliBed and intravenous normal saline is begun at "00 m($h. 9> of the chest
shows a lung lesion that is probably a mass. 9> of the abdomen and pelvis shows an enlarged
liver several nodules suggesting metastatic lesions and bilateral adrenal masses. >he kidneys
appear anatomically normal.
(aboratory studies on hospital day <)
0lood urea nitrogen 3 mg$d(
1erum creatinine ".0 mg$d(
1erum sodium "2* me2$(
1erum potassium *.< me2$(
1erum chloride ," me2$(
1erum bicarbonate 20 me2$(
1erum uric acid *.0 mg$d(
4rinary sodium ""0 me2$(
4rinary potassium <0 me2$(
4rinary creatinine 3% mg$d(
5hich of the following is the most likely cause of the decreasing serum sodium concentration
in this patient6
7A8 =xtracellular fluid volume depletion
708 Addison's disease
798 1yndrome of inappropriate antidiuretic hormone secretion 71FA:!8
7:8 9irrhosis
7=8 9ongestive heart failure
Nephrology - Item 7
A 23-year-old female cafeteria worker is evaluated because of an elevated blood pressure
7approximately "#0$"0% mm !g8 for the past 2 to * months. >he patient feels well. !er
mother has hypertension and kidney disease and a maternal aunt is currently on hemodialysis
because of renal failure. On physical examination her height is "%2 cm 7#2 in8 and weight is
## kg 7"<% lb8. !er blood pressure is "##$"0# mm !g both seated and standing. >he
remainder of the examination is normal.
(aboratory studies)
1erum creatinine 0.3 mg$d(
1erum sodium "<0 me2$(
1erum potassium %.0 me2$(
1erum chloride "02 me2$(
1erum bicarbonate 2% me2$(
1erum thyroid-stimulating hormone ;ormal
4rinalysis ;ormal
An electrocardiogram is normal.
5hich of the following diagnostic studies is most likely to provide information regarding the
cause of her hypertension6
7A8 9aptopril-stimulated renal scan
708 2<-!our urine determination for vanillylmandelic acid
798 Denal ultrasonography
7:8 /lasma renin activity and aldosterone determinations
Nephrology - Item 8
A %0-year-old man underwent perineal prostatectomy " day ago for clinically localiBed prostate
cancer. >he patient has mild hypertension that is well controlled with diltiaBem. !e is otherwise
healthy and preoperative renal function was normal. A @oley catheter was placed
preoperatively. >he patient was in the dorsal lithotomy position for almost # hours and
received approximately * ( of lactated Dinger's solution perioperatively with a concomitant
urine output of %#3 m(. >here was no significant decrease in mean arterial pressure or other
notable events during the procedure. !e tolerated the procedure well with minimal blood loss.
!owever he began to have decreasing urine output postoperatively. @our hours after the
procedure his urine output decreased to "0 m($h. >he catheter was flushed and was found to
be working correctly. >wo fluid boluses of %00 m( of normal saline were administered but
oliguria continued.
On physical examination today the patient is responsive. !is temperature is *#.3 ?9 7,3.2
?@8. !is pulse rate is ,%$min supine and ""0$min sitting respiratory rate is "3$min and blood
pressure is "*2$3% mm !g supine and "20$&3 mm !g sitting. !is neck is supple and no
Gugular venous distention is evident. !is chest is clear. >here is a systolic eGection murmur
heard best at the left sternal border but no gallop is heard. !is abdomen is soft distended
and nontender with infre2uent bowel sounds. >here is no peripheral edema.
(aboratory studies 7immediately after surgery8)
0lood urea nitrogen 20 mg$d(
1erum creatinine 2.2 mg$d(
1erum sodium "<2 me2$(
1erum potassium %.3 me2$(
1erum chloride "0* me2$(
1erum bicarbonate ", me2$(
1erum calcium &.0 mg$d(
1erum phosphorus %., mg$d(
(aboratory studies 72 hours postoperatively8)
0lood urea nitrogen 2* mg$d(
1erum creatinine 2.& mg$d(
1erum sodium "<# me2$(
1erum potassium #., me2$(
1erum chloride "0# me2$(
1erum bicarbonate "< me2$(
4rinalysis 1pecific gravity ".02%. trace protein large blood no glucose. muddy brown
casts and cellular debris
4rinary sodium *2 me2$(
=lectrocardiogram shows sinus tachycardia and peaked > waves.
5hich of the following is the most likely diagnosis6
7A8 Dhabdomyolysis
708 4rinary tract obstruction
798 Acute interstitial nephritis
7:8 Dapidly progressive glomerulonephritis
7=8 /rerenal aBotemia
Nephrology - Item 9
A *"-year-old woman who has had type " diabetes mellitus since she was * years old now has
anasarca proliferative diabetic retinopathy and peripheral sensory neuropathy. !er blood
pressure is "<*$,0 mm !g and her serum creatinine is 2." mg$d( 7serum creatinine was ".<
mg$d( " year ago8.
5hich of the following 2<-hour urinary values is most likely to be found6
7A8 /rotein of #%00 mg$2< h
708 /rotein of 300 mg$2< h
798 /rotein of #& mg$2< h
7:8 Albumin of ""2 mg$2< h
Nephrology - Item 10
A <3-year-old man comes to your office because of a weight gain of "%.% kg 7*< lb8 over the
last * weeks. !e has massive swelling of his feet and legs and reports swelling around his eyes
in the morning and stiffness of his hands upon arising. Medical history is unremarkable.
On physical examination his temperature is *#., ?9 7,3.< ?@8. !is pulse rate is 32$min and
regular respiratory rate is "<$min and blood pressure is "<0$,0 mm !g. /eriorbital edema is
present. >here is no neck vein distention. !is chest is clear. 9ardiac examination is normal. !is
abdomen is soft and nontender. >here is <E pitting edema to both thighs.
(aboratory studies)
!ematocrit <2+
(eukocyte count 3200$-(
/latelet count "3#000$-(
/lasma glucose "2# mg$d(
0lood urea nitrogen "3 mg$d(
1erum creatinine 0., mg$d(
1erum sodium "<0 me2$(
1erum potassium <.* me2$(
1erum chloride "0# me2$(
1erum bicarbonate 2% me2$(
/lasma cholesterol <3# mg$d(
/lasma albumin ".3 g$d(
4rinalysis <E protein several erythrocytes$hpf oval fat bodies
5hich of the following is the most appropriate initial treatment of this patient's edema6
7A8 (ow-salt diet alone with a goal of losing 0.<%-0., kg 7"-2 lb8 daily
708 (ow-salt diet and spironolactone with a goal of losing ".3-2.2 kg 7<-% lb8 daily
798 (ow-salt diet and furosemide with a goal of losing 0.<%-0., kg 7"-2 lb8 daily
7:8 (ow-salt diet and furosemide with a goal of losing ".3-2.2 kg 7<-% lb8 daily
Nephrology - Item 11
Ff the proteinuria of the patient in the preceding 2uestion does not improve which of the
following is most appro-priate for his hyperlipidemia6
7A8 >herapy is not necessary since the hyperlipidemia is not associated with an increased
risk of atherosclerotic events
708 :ietary therapy alone with a low-saturated-fat low-cholesterol diet will usually normaliBe
the plasma cholesterol level
798 >herapy should be started with both a low-saturated-fat low-cholesterol diet and a *-
hydroxy-*-methylglutaryl-coenByme A 7!MC-9oA8 reductase inhibitor
7:8 >he lipoprotein7a8 level should be measured. if elevated therapy with a *-hydroxy-*-
methylglutaryl-coenByme A 7!MC-9oA8 reductase inhibitor should be started
Nephrology - Item 12
A *%-year-old woman has a history of recurrent calcium-containing kidney stones over the
past % years. A previous evaluation included a normal serum calcium concentration and 2<-
hour urinary calcium excretion rate. 1he is seen today because of another episode of right
flank pain. A plain radiograph of the abdomen and pelvis shows a radiopa2ue kidney stone at
the right ureteropelvic Gunction.
(aboratory studies)
0lood urea nitrogen 23 mg$d(
1erum creatinine ".0 mg$d(
1erum sodium "*3 me2$(
1erum potassium *.* me2$(
1erum chloride ""0 me2$(
1erum bicarbonate "3 me2$(
5hich of the following urine studies would be most likely to suggest the correct underlying
disorder6
7A8 9alcium)creatinine ratio
708 >ranstubular potassium gradient
798 p!
7:8 @ractional excretion of sodium
7=8 4rine culture with special instructions to maintain the culture for at least " week
Nephrology - Item 13
A %#-year-old woman is referred to you because of hypertension and hyperlipidemia. 1he had
a thyroidectomy many years ago and has been on thyroid replacement therapy for more than
"% years. @our months ago she was evaluated by her primary care physician because of
fatigue. Multiple blood pressure readings at that time averaged "<#$"0< mm !g. /lasma
cholesterol was *"2 mg$d( and serum triglycerides were 2#3 mg$d(. other routine laboratory
studies were normal. A low-salt low-saturated-fat diet was prescribed but her blood pressure
and hyperlipidemia did not improve. Denal duplex ultrasonography showed a normal-siBed
right kidney with a #0+ stenosis and a normal left kidney and left renal artery. /lasma renin
activity was suppressed at H " ng$( per hour. /lasma catecholamines were slightly elevated at
&%, ng$(. Denal angiography was suggested but the patient asked for a second opinion prior
to having this study done.
On your physical examination her height is "%% cm 7#" in8 and weight is &% kg 7"#% lb8.
/ulse rate is #0$min and blood pressure is "%3$""0 mm !g both seated and standing.
@unduscopic examination shows grade FF hypertensive changes without hemorrhage
exudates or papilledema. A thyroidectomy scar is present. >here are no carotid or peripheral
bruits. 9ardiopulmonary examination is normal. !er abdomen is obese. there are no masses
bruits or enlarged organs. >race bilateral nonpitting pedal edema is noted.
(aboratory studies)
/lasma glucose """ mg$d(
0lood urea nitrogen 22 mg$d(
1erum creatinine 0., mg$d(
1erum sodium "<0 me2$(
1erum potassium <.3 me2$(
1erum chloride ,, me2$(
1erum bicarbonate 23 me2$(
1erum calcium ,.# mg$d(
1erum phosphorus <." mg$d(
A chest radiograph and electrocardiogram are normal.
5hich of the following is most appropriate at this time6
7A8 0egin diuretic therapy
708 /roceed with renal angiography
798 Order thyroid-stimulating hormone determination
7:8 Order 2<-hour urine determination for vanillylmandelic acid. begin b-blocker therapy
Nephrology - Item 14
A 2*-year-old woman comes to the emergency department because of nausea and back pain.
!FI infection was diagnosed 2 months ago. !er serum creatinine concentration at that time
was 0.# mg$d(. !ighly active antiretroviral therapy 7!AAD>8 with stavudine delavirdine and
indinavir was started * weeks ago.
On physical examination the patient is oriented. !er supine pulse rate is ,3$min and regular
and blood pressure is "20$&0 mm !g without orthostatic changes. !er chest is clear and no
cardiac murmur or gallop is audible. Abdominal examination is normal. (eft flank tenderness
and moderate bilateral lower extremity edema are present.
(aboratory studies)
0lood urea nitrogen 30 mg$d(
1erum creatinine <.& mg$d(
1erum sodium "<& me2$(
1erum potassium %.2 me2$(
1erum chloride "00 me2$(
1erum bicarbonate "3 me2$(
4rinalysis 1pecific gravity ".0"#. moderate blood trace protein trace ketones no
glucose. &-"0 erythrocytes$hpf. crystals and tubular cell casts no erythrocyte
casts
5hich of the following is the most appropriate next step in this patient's care6
7A8 :iscontinue indinavir
708 Administer captopril
798 1chedule dialysis
7:8 Administer sodium polystyrene sulfonate 7Jayexalate8
7=8 :iscontinue stavudine
Nephrology - Item 15
A %<-year-old man is evaluated because of new-onset proteinuria detected by dipstick
discovered during an insurance examination. On physical examination in your office he weighs
3" kg 7"&3 lb8 and his blood pressure is "%<$,0 mm !g. >he remainder of his physical
examination is normal.
(aboratory studies)
1erum creatinine "." mg$d(
4rinary volume "020 m($2< h
4rinary creatinine ,"0 mg$2< h
4rinary protein "2%0 mg$2< h
9reatinine clearance #2 m($min
5hich of the following statements is correct regarding the 2<-hour urine collection6
7A8 >he 2<-hour urinary protein is consistent with nephrotic-range proteinuria
708 >he creatinine clearance is inaccurate because the 2<-hour urine collection is incomplete
798 >he creatinine clearance is inaccurate because the presence of proteinuria interferes with
the urinary creatinine determination
7:8 >he 2<-hour urine collection is accurate as evidenced by the fact that most adults excrete
,00 to "000 mg of creatinine daily
Nephrology - Item 16
A <2-year-old woman comes to your office for a routine evaluation. 1he received multiple
transfusions 20 years earlier after an automobile accident. Over the last year her serum
creatinine level has risen from ".0 mg$d( to 2.* mg$d(.
On physical examination her temperature is normal. !er pulse rate is 32$min respiratory rate
is "2$min and blood pressure is "#0$,3 mm !g. !eart sounds are normal and her chest is
clear. Mild hepatosplenomegaly is present. >here is a petechial purpuric rash on her lower
extremities. no edema is present.
(aboratory studies)
9omplete blood count ;ormal
/lasma glucose ;ormal
1erum electrolytes ;ormal
9* 0orderline low normal
9< Deduced
9!%0 Deduced 72%+8
4rinalysis Many erythrocytes$hpf. erythrocyte casts
2<-!our urinary protein *.% g$2< h
5hich of the following additional laboratory studies are most appropriate6
7A8 Assays for antineutrophil cytoplasmic antibodies 7A;9A8 and anti-glomerular basement
membrane 7anti-C0M8 antibodies
708 1erum protein electrophoresis and urine immuno-electrophoresis
798 Antistreptolysin O titer and antihyaluronidase assay
7:8 9ryoglobulin determination and serologic tests for hepatitis 9
Nephrology - Item 17
A 23-year-old pregnant woman with a %-year history of proteinuria associated with focal and
segmental glomerulo-sclerosis is seen 2 months after her last menstrual period. @ive years
ago her serum creatinine concentration was ".0 mg$d( and her creatinine clearance was 30
m($min. >hree years ago her serum creatinine concentration was ".< mg$d(. 1he was treated
with fosinopril with good blood pressure control but the drug was discontinued " month ago
and methyldopa was started. On physical examination she has gained 2.& kg 7# lb8. !er blood
pressure is "*%$33 mm !g. 1he has trace pedal edema.
(aboratory studies)
0lood urea nitrogen 23 mg$d(
1erum creatinine 2.2 mg$d(
1erum uric acid %." mg$d(
4rinalysis <E protein
5hich of the following statements about this patient's course is correct6
7A8 1he is not at high risk for developing preeclampsia
708 1he is not at high risk for a premature birth
798 1he has a "+ to 2+ risk of developing more severe renal insufficiency
7:8 !er fetus is at risk for intrauterine growth retardation
Nephrology - Item 18
A #2-year-old male engineer is referred to you because of his difficult-to-control hypertension.
!igh blood pressure was first noted "0 months ago and has not responded to increasing doses
of amlodipine losartan and hydro-chlorothiaBide$triamterene. !e has no history of cardio-
vascular or renal disease. Denal function studies urinalysis and renal scan obtained by his
referring physician were normal.
On physical examination his height is "30 cm 7&2 in8 and weight is 32 kg 7"30 lb8. 0lood
pressure is "&0$"0% mm !g seated and standing. @unduscopic examination shows grade FF
hypertensive retinopathy. 9ardiopulmonary examination is normal. Abdominal examination
discloses no masses bruits or enlarged organs. >race bilateral pedal edema is noted.
(aboratory studies)
/lasma glucose 3, mg$d(
0lood urea nitrogen "# mg$d(
1erum creatinine 0.& mg$d(
1erum sodium "<0 me2$(
1erum potassium *.2 me2$(
1erum chloride "00 me2$(
1erum bicarbonate 2& me2$(
1erum thyroid-stimulating hormone ;ormal
4rinalysis ;ormal
5hich of the following would be most appropriate at this time6
7A8 Add potassium chloride to his current regimen to correct his hypokalemia. no additional
diagnostic studies are indicated
708 :iscontinue losartan and hydrochlorothiaBide$triamterene for 2 weeks. obtain a 2<-hour
urine specimen for sodium potassium and aldosterone excretion during oral salt-loading
798 :iscontinue losartan. order a captopril-stimulated renal scan
7:8 :iscontinue losartan and hydrochlorothiaBide$triamterene for 2 weeks. obtain a serum
aldosterone determination
Nephrology - Item 19
A %3-year-old woman is hospitaliBed because of acute renal failure. One week ago acute 0-
cell lymphoblastic leukemia was diagnosed. >oday * days before she was to begin
chemotherapy she came to the office for examination and to receive a prescription for
allopurinol. 1he was asymptomatic but stated that her urine output had been decreasing for
2< to *# hours.
On physical examination her temperature was *&.*?9 7,,.2 ?@8. !er pulse rate was "00$min
and regular respiratory rate was "<$min and regular and blood pressure was "20$#0 mm !g
without orthostatic changes. :iffuse lymphadenopathy was noted. 9ardiovascular examination
was normal. >he liver and spleen were not enlarged and there was no abdominal bruit.
;eurologic examination was normal. :istal pulses were strong and present bilaterally. >here
was "E pedal edema.
(aboratory studies)
!ematocrit 2,+
(eukocyte count 2<%00$-(. %2+ lymphocytes %+ eosinophils "+ neutrophils *+
monocytes 2+ basophils *&+ large undetermined cells
/latelet count ,,000$-(
/rothrombin time ;ormal
0lood urea nitrogen <* mg$d(
1erum creatinine *.0 mg$d(
1erum sodium "<< me2$(
1erum potassium %.# me2$(
1erum chloride "00 me2$(
1erum bicarbonate "% me2$(
1erum calcium ,.2 mg$d(
1erum phosphorus <.* mg$d(
1erum uric acid ",.* mg$d(
1erum aspartate aminotransferase <0 4$(
1erum alanine aminotransferase *% 4$(
1erum amylase <, 4$(
1erum bilirubin 0., mg$d(
4rinalysis 1pecific gravity ".00& p! %.0. "E protein no blood ketones or glucose. rare
leukocytes$hpf several granular cell casts no leukocyte or erythrocyte casts
4rinary sodium <# me2$(
4rinary osmolality 203 mosm$kg !2O
4rinary creatinine 2% mg$d(
4rinary uric acid *& mg$d(
>he patient is hospitaliBed. A 2<-hour urine collection after admission to the hospital showed a
volume of *%% m(. 4rine culture showed no growth.
5hich of the following statements is correct regarding acute renal failure in this patient6
7A8 >he fractional excretion of sodium suggests prerenal aBotemia
708 >he urinary uric acid)urinary creatinine ratio suggests that uric acid-mediated inGury
underlies the pathologic process
798 >he course is consistent with rhabdomyolysis
7:8 >he absence of hyperphosphatemia rules out the diagnosis of tumor lysis syndrome
7=8 >umor lysis syndrome does not develop unless antineoplastic therapy is administered
Nephrology - Item 20
A %3-year-old woman is hospitaliBed because of acute renal failure. One week ago acute 0-
cell lymphoblastic leukemia was diagnosed. >oday * days before she was to begin
chemotherapy she came to the office for examination and to receive a prescription for
allopurinol. 1he was asymptomatic but stated that her urine output had been decreasing for
2< to *# hours.
On physical examination her temperature was *&.*?9 7,,.2 ?@8. !er pulse rate was "00$min
and regular respiratory rate was "<$min and regular and blood pressure was "20$#0 mm !g
without orthostatic changes. :iffuse lymphadenopathy was noted. 9ardiovascular examination
was normal. >he liver and spleen were not enlarged and there was no abdominal bruit.
;eurologic examination was normal. :istal pulses were strong and present bilaterally. >here
was "E pedal edema.
(aboratory studies)
!ematocrit 2,+
(eukocyte count 2<%00$-(. %2+ lymphocytes %+ eosinophils "+ neutrophils *+
monocytes 2+ basophils *&+ large undetermined cells
/latelet count ,,000$-(
/rothrombin time ;ormal
0lood urea nitrogen <* mg$d(
1erum creatinine *.0 mg$d(
1erum sodium "<< me2$(
1erum potassium %.# me2$(
1erum chloride "00 me2$(
1erum bicarbonate "% me2$(
1erum calcium ,.2 mg$d(
1erum phosphorus <.* mg$d(
1erum uric acid ",.* mg$d(
1erum aspartate aminotransferase <0 4$(
1erum alanine aminotransferase *% 4$(
1erum amylase <, 4$(
1erum bilirubin 0., mg$d(
4rinalysis 1pecific gravity ".00& p! %.0. "E protein no blood ketones or glucose. rare
leukocytes$hpf several granular cell casts no leukocyte or erythrocyte casts
4rinary sodium <# me2$(
4rinary osmolality 203 mosm$kg !2O
4rinary creatinine 2% mg$d(
4rinary uric acid *& mg$d(
>he patient is hospitaliBed. A 2<-hour urine collection after admission to the hospital showed a
volume of *%% m(. 4rine culture showed no growth.
5hich of the following is contraindicated in the initial management of this patient6
7A8 4ltrasonography or non-contrast-enhanced 9> of the abdomen
708 Administration of allopurinol
798 :ialysis as needed
7:8 Denal arteriography
Nephrology - Item 21
A <2-year-old man is evaluated because of right renal colic and microhematuria. !e has had
three previous episodes of calcium nephrolithiasis. :iagnostic studies show a <-mm calcified
stone in the middle right ureter.
5hich of the following laboratory test results is not a risk factor for calcium nephrolithiasis6
7A8 4rinary calcium of *"% mg$2< h
708 1erum uric acid of "0.% mg$d(
798 4rinary citrate of "00 mg$2< h 7normal *00-&00 mg$2< h8
7:8 4rinary oxalate of &2 mg$2< h 7normal H <0 mg$2< h8
7=8 1erum calcium of "" mg$d( and serum phosphorus of 2." mg$d(
Nephrology - Item 22
A %<-year-old black American man is found to have idiopathic nephrotic syndrome with normal
blood urea nitrogen and serum creatinine levels. A renal biopsy has been recommended to
determine the histopathologic diagnosis and potential course of therapy. 0efore undergoing
the biopsy the patient wishes to know what is most likely to be found and whether it is
treatable.
5hich of the following is the most appropriate answer to this patient's 2uestion6
7A8 Membranous nephropathy is the most likely diagnosis and immunosuppressive therapy is
successful in up to %0+ of patients
708 Minimal change disease is the most likely diagnosis because of the normal blood urea
nitrogen and serum creatinine levels and treatment with glucocorticoids will likely induce a
remission of the proteinuria
798 @ocal glomerulosclerosis is the most likely diagnosis because it is the most common
pattern of idiopathic nephrotic syndrome in black Americans and treatment with a prolonged
course of glucocorticoids is successful in up to %0+ of patients
7:8 A( amyloidosis is the most likely diagnosis in patients over %0 years of age and
treatment includes chemotherapy
Nephrology - Item 23
A 2#-year-old woman comes to the emergency department because of paresthesias perioral
numbness and generaliBed weakness. 1he denies use of any medications. !er blood pressure
is "20$33 mm !g and physical examination is unremarkable.
(aboratory studies)
0lood urea nitrogen "% mg$d(
1erum creatinine ".2 mg$d(
1erum sodium "*# me2$(
1erum potassium 2.3 me2$(
1erum chloride ,0 me2$(
1erum bicarbonate *3 me2$(
4rine p! %.3
4rinary sodium "3 me2$(
4rinary potassium 20 me2$(
4rinary chloride % me2$(
4rinary calcium)creatinine ratio 0.*
5hich of the following is the most likely diagnosis6
7A8 0artter's syndrome
708 1urreptitious vomiting
798 Citelman's syndrome
7:8 !ypokalemic periodic paralysis
7=8 (icorice ingestion
Nephrology - Item 24
A <3-year-old woman was found to have primary hypertension # months ago. :espite a trial of
lifestyle modifications her blood pressure remained elevated at about "%3$,# mm !g.
>herapy with amlodipine % mg daily was begun.
>he patient returns for a follow-up visit # weeks after beginning amlodipine. 1everal blood
pressures readings in your office average "%2$,2 mm !g. 1he has also noted progressive
ankle edema since therapy was begun.
5hich of the following is most appropriate at this time6
7A8 ;o change in therapy. return for a follow-up visit in * to # weeks
708 9hange to another antihypertensive agent and increase the dose until her blood pressure
is H "<0$,0 mm !g
798 Fncrease the amlodipine to "0 mg daily
7:8 /rescribe a low-salt diet and support hose for control of localiBed edema
Nephrology - Item 25
A %,-year-old male smoker is hospitaliBed after sustaining an acute anterior wall myocardial
infarction. !e has a "#-year history of type 2 diabetes mellitus and hypertension. 9urrent
medications are metformin and diltiaBem.
On physical examination his temperature is *&.3 ?9 7"0".3 ?@8. !is pulse rate is &%$min
respiratory rate is "3$min and blood pressure is "<%$3% mm !g. Mucous membranes are dry
and moderate Gugular venous distention is present. !is chest is clear and no murmurs or
gallops are heard. !is abdomen is soft nontender and nondistended. >here is no sacral or
lower extremity edema.
(aboratory studies 7on admission8)
!emoglobin "2.0 g$d(
!ematocrit *3+
(eukocyte count 3000$-(
/latelet count 2%#000$-(
0lood urea nitrogen 22 mg$d(
1erum creatinine ".# mg$d(
1erum sodium "<0 me2$(
1erum potassium %." me2$(
1erum chloride "0% me2$(
1erum bicarbonate 2# me2$(
1erum calcium ,.0 mg$d(
1erum magnesium ".# mg$d(
1erum phosphorus 2., mg$d(
4rinalysis p! #.0. "E protein "E glucose no blood or ketones. no formed
elements on microscopic examination
>he patient is scheduled to undergo coronary angiography in the morning.
5hich of the following is the most effective preventive measure to reduce the risk of contrast
nephropathy in this patient6
7A8 Administer intravenous half-normal saline and intravenous furosemide at the time of
angiography
708 Administer intravenous half-normal saline and intravenous mannitol at the time of
angiography
798 Administer intravenous half-normal saline before performing angiography
7:8 (imit the dose of intravenous contrast to H 0.% m($kg$h and administer concurrent
intravenous dopamine at the time of angiography
7=8 :iscontinue the diltiaBem
Nephrology - Item 26
A %,-year-old male smoker is hospitaliBed after sustaining an acute anterior wall myocardial
infarction. !e has a "#-year history of type 2 diabetes mellitus and hypertension. 9urrent
medications are metformin and diltiaBem. On physical examination his temperature is *&.3 ?9
7"0".3 ?@8. !is pulse rate is &%$min respiratory rate is "3$min and blood pressure is "<%$3%
mm !g. Mucous membranes are dry and moderate Gugular venous distention is present. !is
chest is clear and no murmurs or gallops are heard. !is abdomen is soft nontender and
nondistended. >here is no sacral or lower extremity edema.
(aboratory studies 7on admission8)
!emoglobin "2.0 g$d(
!ematocrit *3+
(eukocyte count 3000$-(
/latelet count 2%#000$-(
0lood urea nitrogen 22 mg$d(
1erum creatinine ".# mg$d(
1erum sodium "<0 me2$(
1erum potassium %." me2$(
1erum chloride "0% me2$(
1erum bicarbonate 2# me2$(
1erum calcium ,.0 mg$d(
1erum magnesium ".# mg$d(
1erum phosphorus 2., mg$d(
4rinalysis p! #.0. "E protein "E glucose no blood or ketones. no formed
elements on microscopic examination
Angiography revealed two lesions one in the proximal left anterior descending artery and one
in the first diagonal artery. 0oth were treated with angioplasty and stent placement.
>hirty-six hours after the procedure the patient's laboratory values were)
0lood urea nitrogen ** mg$d(
1erum creatinine 2., mg$d(
1erum sodium "<* me2$(
1erum potassium %.* me2$(
1erum chloride ""0 me2$(
1erum bicarbonate 2< me2$(
4rinalysis 1pecific gravity ".0"%. trace protein trace glucose no blood. few
hyaline casts on microscopic examination
5hich of the following is indicated to reduce the morbidity associated with the patient's
contrast nephropathy6
7A8 Monitor. begin renal replacement therapy if indicated
708 Administer intravenous half-normal saline and intravenous dopamine for 2< hours
798 Administer intravenous half-normal saline
7:8 0egin dialysis to clear residual contrast as soon as possible
7=8 :iscontinue the diltiaBem
Nephrology - Item 27
>hree months ago a <&-year-old woman experienced mild right flank pain for several hours
associated with gross hematuria. 1ince then she has had several episodes of gross painless
hematuria. >here has been no fever chills dysuria or weight loss.
On physical examination she is alert and in no distress. >emperature is *#.# ?9 7,&., ?@8. !er
pulse rate is #,$min and blood pressure is "*&$3, mm !g. >he examination is normal except
for moderate obesity and slight costovertebral angle tenderness.
(aboratory studies)
1erum creatinine 0.3 mg$d(
1erum sodium "<0 me2$(
1erum potassium *.3 me2$(
1erum chloride "02 me2$(
1erum bicarbonate 2# me2$(
4rinalysis p! <.%. *E heme no protein. <0-%0 erythrocytes occasional
leukocytes$hpf. no bacteria no crystals
A plain radiograph of the abdomen and pelvis is normal. 1piral 9> without contrast shows a
bright ".%-cm obGect in the right renal pelvis with slight right hydronephrosis. >he kidneys are
"2 cm in length with normal parenchymal thickness.
5hich of the following is the most likely diagnosis6
7A8 9alcium oxalate stone in the right renal pelvis
708 >ransitional cell carcinoma of the bladder
798 4ric acid stone in the right renal pelvis
7:8 Analgesic nephropathy
7=8 9hronic pyelonephritis
Nephrology - Item 28
A #3-year-old man is evaluated because of progressive swelling of his lower extremities. !e
has no prior history of cardiovascular or renal disease.
On physical examination his temperature is normal. !is pulse rate is #3$min respiratory rate
is "2$min and blood pressure is "*3$32 mm !g. !is chest is clear. 9ardiac examination
reveals an 1< gallop. Abdominal examination discloses a palpable spleen tip and an enlarged
liver. >here is <E bilateral pedal edema.
(aboratory studies)
9omplete blood count ;ormal
/lasma glucose ;ormal
0lood urea nitrogen 2% mg$d(
1erum creatinine ".3 mg$d(
1erum electrolytes ;ormal
4rinalysis *E to <E protein trace heme
2<-!our urinary albumin %.# g$2< h
An electrocardiogram shows low voltage but no other abnormalities. A chest radiograph is
normal. 1erum protein electrophoresis discloses an abnormal monoclonal protein in the
gamma region which on immunoelectrophoresis is found to be 2.* g of FgC lambda.
5hich of the following is the most likely cause of this patient's renal insufficiency6
7A8 Membranous nephropathy
708 Myeloma cast nephropathy
798 Amyloidosis
7:8 (ight chain deposition disease
Nephrology - Item 29
A %0-year-old woman with a long history of type " diabetes mellitus develops an upper
respiratory tract infection and fever. 1he then develops polyuria and thirst and comes to the
emergency department. On physical examination her temperature is *3.* ?9 7"0".0 ?@8. !er
blood pressure is ""0$&0 mm !g supine and ,0$#% mm !g seated. @unduscopic examination
shows diabetic retinopathy. 9ardiopulmonary examination is normal. Abdominal examination
reveals some right upper 2uadrant tenderness. >here is "E peripheral edema.
(aboratory studies)
/lasma glucose "200 mg$d(
0lood urea nitrogen *3 mg$d(
1erum creatinine 2." mg$d(
1erum sodium "2< me2$(
1erum potassium <." me2$(
1erum chloride ,0 me2$(
1erum bicarbonate "0 me2$(
A diagnosis of ketoacidosis is made. Along with insulin isotonic saline is infused to restore
extracellular fluid volume and potassium chloride is given to correct the hypokalemia that
later developed. ;o free water or additional solute was administered.
>his patient's serum sodium concentration is most likely to be which of the following after her
plasma glucose has been restored to "20 mg$d(6
7A8 (ess than "20 me2$(
708 About "2< me2$(
798 "*% to "<% me2$(
7:8 Creater than "<, me2$(
Nephrology - Item 30
A %%-year-old male lawyer has progressive chronic renal insufficiency due to type 2 diabetic
nephropathy and hypertension. !is creatinine clearance is 2* m($min and his serum
creatinine is *." mg$d(. !e has Gust returned from an introductory educational session
regarding dialysis and transplantation options which was arranged by the consulting
nephrologist. !e asks your opinion about potential options.
5hich of the following offers the best prognosis for this patient6
7A8 /eritoneal dialysis
708 !emodialysis
798 Denal transplantation
7:8 9ombined renal and pancreas transplantation
Nephrology - Item 31
A %*-year-old 9hinese-American woman recently returned to the 4nited 1tates from a trip to
9hina. >ype 2 diabetes mellitus was diagnosed < months before the start of her trip. 0ecause
she was at her ideal weight treatment with oral hypoglycemics was suggested but she
declined because she was Kafraid of medicines.K 1he was normotensive and her serum
creatinine concentration at the time of diagnosis was 0.& mg$d(. >here was no evidence of
microalbuminuria. On her return to the 4nited 1tates she saw her primary care physician for
her regular checkup. !er physical examination was unchanged but her serum creatinine
concentration was "., mg$d(. On 2uestioning she told her physician that she had started
taking medications in 9hina Kto protect her kidneys.K 1he is referred to you.
On physical examination her temperature is *#.3 ?9 7,3.2 ?@8. !er pulse rate is 33$min
respiratory rate is "<$min and regular and blood pressure is "*0$32 mm !g without
orthostatic changes. >here is no rash and no neck vein distention. !er chest is clear. 1" and
12 are normal and no gallops or murmurs are heard. Abdominal examination is normal. >here
is "E pedal edema. :istal pulses are present. ;eurologic examination is normal.
(aboratory studies)
!ematocrit 23+
(eukocyte count #%00$-(. normal differential
/latelet count 2,,000$-(
0lood urea nitrogen *< mg$d(
1erum creatinine "., mg$d(
1erum sodium "<" me2$(
1erum potassium %.* me2$(
1erum chloride ,, me2$(
1erum bicarbonate 2" me2$(
1erum bilirubin 0., mg$d(
1erum aspartate aminotransferase ;ormal
1erum alanine aminotransferase ;ormal
1erum amylase ;ormal
1erum creatine kinase ;ormal
4rinalysis 1pecific gravity ".00& p! %.0. "E protein no blood ketones or glucose. no
erythrocytes scant leukocytes$hpf several granular casts no erythrocyte or
leukocyte casts
4rine culture ;o growth
5hich of the following is the most likely cause of the acute renal failure6
7A8 :iabetic nephropathy
708 FgA nephropathy
798 ;ephropathy due to 9hinese herbs
7:8 /rerenal aBotemia
Nephrology - Item 32
A *<-year-old woman is referred from the emergency department because of bilateral foot and
leg swelling. 1he recently was found to be infected with !FI.
On physical examination her temperature is *#.& ?9 7,3.0 ?@8. !er pulse rate is 33$min
respiratory rate is "<$min and blood pressure is "2#$&3 mm !g. >here is 2E to *E pedal and
ankle edema. >he remainder of the physical examination is normal.
(aboratory studies)
!ematocrit *%+
(eukocyte count %#00$-(
/latelet count *<%000$-(
0lood urea nitrogen 22 mg$d(
1erum creatinine ".# mg$d(
1erum total complement ;ormal
Antinuclear antibody ;egative
Antistreptolysin O ;egative
Antineutrophil cytoplasmic antibody ;egative
4rinalysis <E protein 2E heme. several erythrocytes$hpf broad casts
2<-!our urinary protein "# g$2< h
5hich of the following diagnostic studies may help suggest the most likely cause of this
patient's renal disorder prior to renal biopsy6
7A8 Denal ultrasonography
708 Fntravenous pyelography
798 Denal angiography
7:8 =chocardiography
Nephrology - Item 33
A renal biopsy is performed on the patient in the preceding 2uestion and shows collapsing
focal glomerulosclerosis with tubular microcyst formation.
Fn addition to highly active antiretroviral therapy 7!AAD>8 for the !FI infection which of the
following is the most appropriate therapy for this patient's renal disease6
7A8 An angiotensin-converting enByme inhibitor
708 /ulse intravenous methylprednisolone
798 A dihydropyridine calcium channel blocker
7:8 9yclophosphamide
Nephrology - Item 34
A *%-year-old man with chronic renal failure undergoes hemodialysis three times each week.
!is compliance has been poor and he has missed his last dialysis treatment. !e now comes to
the emergency department because of weakness and nausea.
(aboratory studies)
1erum sodium "23 me2$(
1erum potassium &.2 me2$(
1erum chloride ,% me2$(
1erum bicarbonate "% me2$(
An electrocardiogram shows first-degree heart block peaked > waves and wide LD1
complexes. 5hile awaiting dialysis he receives infusions of calcium gluconate sodium
bicarbonate glucose and insulin. !e also receives inhaled albuterol and oral sodium
polystyrene sulfonate 7Jayexalate8.
5hich of these medications is least likely to reduce his serum potassium concentration6
7A8 Albuterol
708 1odium bicarbonate
798 Clucose and insulin
7:8 9alcium gluconate
7=8 1odium polystyrene sulfonate 7Jayexalate8
Nephrology - Item 35
A 2,-year-old woman with type " diabetes mellitus is evaluated because of a "0-day history of
urinary fre2uency mild dysuria and Kslight fever.K 1he has no history of chills hema-turia
renal stones or flank pain. 1he reports that similar episodes have occurred once or twice
yearly for the past # years. 1he previously treated herself with fluids and cranberry Guice or
went to an urgent care center where she was told that she had cystitis and was given
antibiotics. >here is no family history of renal disease. /hysical examination is normal.
(aboratory studies)
!ematocrit *,+
1erum creatinine 0.3 mg$d(
1erum sodium "*, me2$(
1erum potassium <.0 me2$(
1erum chloride "00 me2$(
1erum bicarbonate 2< me2$(
1erum calcium ,.0 mg$d(
1erum albumin 2.3 g$d(
4rinalysis p! #.%. 2E heme trace protein trace glucose. %-"0 erythrocytes <0-
%0 leukocytes$hpf. many bacteria. leukocyte-esterase positive
4rine culture M "00000 colonies of /roteus mirabilis
A plain radiograph of the abdomen and pelvis shows an irregular calcified obGect measuring *
N % cm overlying the right renal shadow. An intravenous pyelogram shows a branched calcified
calculus occupying the upper half of the right renal collecting system with loss of renal
parenchymal thickness of the right kidney. >he left kidney is normal.
5hich of the following stone diseases does this patient most likely have6
7A8 9alcium oxalate stone
708 9alcium phosphate and oxalate stones
798 4ric acid stone
7:8 >riple phosphate 7struvite8 stone
Nephrology - Item 36
A <3-year-old male renal transplant recipient is evaluated because of severe pain in his right
great toe for the past *# hours. !e denies fever chills or other systemic symptoms. >he
patient had end-stage renal disease secondary to autosomal dominant polycystic kidney
disease and received a successful renal transplant 2 years ago. !is serum creatinine level has
remained stable at 2." mg$d( on an immunosuppressive regimen of prednisone &.% mg daily.
aBathioprine "00 mg daily. and cyclosporine "2% mg twice daily. >he patient has a history of
intermittent gout 7documented by arthrocentesis8 which has flared more fre2uently during the
past # months.
On physical examination his height is "#% cm 7#% in8 and weight is &2 kg 7"%, lb8. !is blood
pressure is "*3$33 mm !g seated and standing. =xamination is normal except for ex2uisite
tenderness and erythema at the base of his right great toe.
(aboratory studies)
9omplete blood count ;ormal
/lasma glucose """ mg$d(
0lood urea nitrogen *0 mg$d(
1erum creatinine "., mg$d(
1erum sodium "*3 me2$(
1erum potassium <.3 me2$(
1erum chloride "0" me2$(
1erum bicarbonate 2* me2$(
1erum uric acid "0." mg$d(
4rinalysis ;ormal
5hich of the following is most appropriate for treating this patient's gout6
7A8 0egin indomethacin 2% mg to %0 mg three times daily and allopurinol *00 mg daily
708 0egin colchicine. recommend subse2uent allopurinol *00 mg daily
798 Fncrease prednisone to *0 mg daily with a tapering dose. recommend subse2uent
allopurinol *00 mg daily
7:8 0egin colchicine. recommend subse2uent allopurinol "00 mg daily with a %0+ to &%+
reduction of his aBathioprine dose
Nephrology - Item 37
A %%-year-old woman was hospitaliBed because of fever and fatigue. Medical history revealed
mitral valve prolapse and mild essential hypertension that has been well controlled with
atenolol for the past # years. 0lood cultures after admission grew viridans streptococci. A
diagnosis of bacte-rial endocarditis was made and intravenous gentamicin and ampicillin were
begun. 1erum creatinine on admission was 0.# mg$d(. On day 3 the patient was discharged
and placed on home intravenous therapy. !er serum creatinine on discharge was 0.& mg$d(.
On day "# of therapy her serum creatinine drawn by the visiting nurse was 2.2 mg$d(. 1he
is readmitted to the hospital.
On physical examination on admission temperature is *#.3 ?9 7,3.2 ?@8. !er pulse rate is
3&$min respiratory rate is "2$min and blood pressure is "#&$,3 mm !g without orthostatic
changes. Mucous membranes are moist and there is no rash. >here is no Gugular venous
distention. !er chest is clear. >here is a soft systolic murmur at the apex but no gallop is
heard. !er abdomen is soft and nontender. (ower extremity pulses are 2E and there is no
sacral or lower extremity edema.
(aboratory studies)
!emoglobin "".# g$d(
!ematocrit *%+
(eukocyte count 3%00$-(. #%+ neutrophils *0+ lymphocytes *+ monocytes 2+ basophils
/latelet count "%2000$-(
0lood urea nitrogen *, mg$d(
1erum creatinine 2.* mg$d(
1erum sodium "<* me2$(
1erum potassium %.% me2$(
1erum chloride "0, me2$(
1erum bicarbonate 2" me2$(
1erum calcium 3.& mg$d(
1erum phosphorus <.& mg$d(
4rinalysis 1pecific gravity ".0"". trace protein no blood or glucose. muddy
granular casts but no erythrocyte or leukocyte casts. no bacteria
4rinary sodium <2 me2$(
5hich of the following is the most likely diagnosis6
7A8 /rerenal aBotemia
708 Acute interstitial nephritis
798 Aminoglycoside nephrotoxicity
7:8 Dhabdomyolysis
7=8 Clomerulonephritis associated with bacterial endocarditis
Nephrology - Item 38
A %%-year-old woman was hospitaliBed because of fever and fatigue. Medical history revealed
mitral valve prolapse and mild essential hypertension that has been well controlled with
atenolol for the past # years. 0lood cultures after admission grew viridans streptococci. A
diagnosis of bacte-rial endocarditis was made and intravenous gentamicin and ampicillin were
begun. 1erum creatinine on admission was 0.# mg$d(. On day 3 the patient was discharged
and placed on home intravenous therapy. !er serum creatinine on discharge was 0.& mg$d(.
On day "# of therapy her serum creatinine drawn by the visiting nurse was 2.2 mg$d(. 1he
is readmitted to the hospital. On physical examination on admission temperature is *#.3 ?9
7,3.2 ?@8. !er pulse rate is 3&$min respiratory rate is "2$min and blood pressure is "#&$,3
mm !g without orthostatic changes. Mucous membranes are moist and there is no rash.
>here is no Gugular venous distention. !er chest is clear. >here is a soft systolic murmur at the
apex but no gallop is heard. !er abdomen is soft and nontender. (ower extremity pulses are
2E and there is no sacral or lower extremity edema.
(aboratory studies)
!emoglobin "".# g$d(
!ematocrit *%+
(eukocyte count 3%00$-(. #%+ neutrophils *0+ lymphocytes *+ monocytes 2+ basophils
/latelet count "%2000$-(
0lood urea nitrogen *, mg$d(
1erum creatinine 2.* mg$d(
1erum sodium "<* me2$(
1erum potassium %.% me2$(
1erum chloride "0, me2$(
1erum bicarbonate 2" me2$(
1erum calcium 3.& mg$d(
1erum phosphorus <.& mg$d(
4rinalysis 1pecific gravity ".0"". trace protein no blood or glucose. muddy granular
casts but no erythrocyte or leukocyte casts. no bacteria
4rinary sodium <2 me2$(
5hich of the following is most appropriate at this time6
7A8 0egin intravenous methylprednisolone
708 0egin intravenous normal saline with dopamine at * units$kg$min
798 :iscontinue gentamicin
7:8 AlkaliniBe the urine with sodium bicarbonate
Nephrology - Item 39
5hich of the following statements is correct regarding autosomal dominant polycystic kidney
disease 7A:/J:86
7A8 A:/J: is invariably associated with end-stage renal disease by the time patients are %0
years old
708 A:/J: is more common in men than in women
798 A:/J: is associated with hepatic cysts in up to %0+ of patients as they age
7:8 A:/J: is less common in black Americans than in other racial groups in the 4nited
1tates
Nephrology - Item 40
A 22-year-old female college student comes to your office because she developed dark urine
after playing basketball. 1he denies Goint symptoms or rash but does recall a sore throat about
" week prior to the episode of dark urine. On physical examination her blood pressure is
normal and there is no edema. >he remainder of the examination is normal. 9omplete blood
count blood urea nitrogen serum creatinine serum electrolytes and serum complement are
also normal. Antinuclear antibody assay is negative. 4rinalysis shows "E protein many
erythrocytes$hpf and erythrocyte casts.
5hich of the following is the most likely diagnosis6
7A8 /oststreptococcal glomerulonephritis
708 (upus nephritis
798 FgA nephropathy
7:8 1ickle cell nephropathy
Nephrology - Item 41
A 2<-year-old man is brought to the emergency department after being found unresponsive on
the floor of his shower. !e has a longstanding history of chronic schiBophrenia and has
re2uired two previous hospitaliBations for hyponatremia related to psychogenic water
ingestion.
On physical examination he is still unresponsive. !e has bitten his tongue and may be
postictal. !is pulse rate is "00$min and regular and blood pressure is "*0$30 mm !g. /hysical
examination is otherwise unremarkable.
(aboratory studies)
0lood urea nitrogen "2 mg$d(
1erum creatinine 0.3 mg$d(
1erum sodium "03 me2$(
1erum potassium *.% me2$(
1erum chloride 30 me2$(
1erum bicarbonate "# me2$(
5hich of the following is most appropriate for this patient6
7A8 !ypertonic 7*+8 saline to increase the sodium concentration by "0 to "% me2$( over
several hours
708 !ypertonic 7*+8 saline to increase the sodium con-centration by 0.% me2$h over 2<
hours
798 /arenteral furosemide and strict water restriction 7H 200 m($2< h8
7:8 ;ormal saline 2 ( at 200 m($h
Nephrology - Item 42
A %<-year-old male renal transplant recipient is evaluated because of headache diBBiness and
loss of balance. >here is no history of loss of consciousness or seiBures.
>he patient has end-stage renal disease secondary to chronic glomerulonephritis. After a
period of hemodialysis he received a successful cadaveric renal transplant "* months ago. !is
course was complicated by two bouts of acute reGection re2uiring intravenous
methylprednisolone and muromonab 9:* 7OJ>*8 therapy. After his first bout of reGection he
developed cytomegalovirus infection. 9urrent medications are prednisone cyclosporine and
aBathioprine. !is serum creatinine level has been stable on this regimen.
On physical examination he appears well. 0lood pressure is "<2$3# mm !g without
orthostatic changes. ;eurologic examination discloses mild nystagmus and a positive Domberg
sign. 9ardiopulmonary and abdominal examinations are normal. >here is no peripheral
lymphadenopathy.
MDF of the head shows a *-cm lesion in the posterior fossa with a patent fourth ventricle.
0iopsy of the mass shows that the lesion is a large cell lymphoma 7non-!odgkin's lymphoma8.
5hich of the following statements is correct regarding this patient's disorder6
7A8 >his disorder is much more common in renal transplant recipients than in heart
transplant recipients because of the increased immunosuppressive therapy re2uired
708 >his disorder is usually due to a !odgkin's lymphoma of >-cell origin that develops in
nodal rather than extranodal sites
798 >he appearance of this disorder is unrelated to the type of organ transplanted and the
immunosuppressive therapy used
7:8 >his disorder appears to be related to 0-cell proliferation induced by =pstein-0arr virus
infection in patients who are receiving chronic immunosuppressive therapy
Nephrology - Item 43
A #*-year-old woman who has chronic inflammatory demyelinating polyneuropathy was
started on intravenous immune globulin C 7FgC8 after having had no response to prolonged
glucocorticoid therapy. Clucocorticoids were stopped 2 months ago. !er first dose of immune
globulin C was & days ago and she received four additional doses of <00 mg$kg ending 2
days ago. 1he experienced headaches during some of the infusions and was treated with
ibuprofen on the third and fourth days. 1he did not notice a change in her neurologic status
after therapy. 1he contacts you today because of scanty urine output and discolored urine over
the last 2< hours. >he patient has no history of diabetes mellitus hypertension or renal
disease.
On physical examination her temperature is *&.2 ?9 7,,.0 ?@8. !er pulse rate is "20$min
respiratory rate is 20$min and blood pressure is "#0$,3 mm !g. 1cant crackles are
auscultated but no murmurs or gallops are heard. Abdominal examination is normal. >here is
2E peripheral edema. ;eurologic examination shows severe distal lower extremity weakness
areflexia and markedly diminished sensation.
(aboratory studies)
0lood urea nitrogen *0 mg$d(
1erum creatinine 2.& mg$d(
1erum sodium "<2 me2$(
1erum potassium %." me2$(
1erum chloride ,3 me2$(
1erum bicarbonate "# me2$(
4rinalysis 1pecific gravity ".00,. "E protein. 2-* erythrocytes "0 leukocytes$hpf. no
erythrocyte or leukocyte casts
4rine culture ;o growth
5hich of the following is most appropriate at this time6
7A8 /rovide vigorous fluid repletion
708 /repare the patient for chronic dialysis
798 /erform plasmapheresis
7:8 :iscontinue immune globulin C
Nephrology - Item 44
A *3-year-old man with chronic renal failure comes to the emergency department because of
weakness anorexia and nausea. !e has recently had increasing nausea and vomiting but has
refused to begin dialysis despite your recommendation to do so. /hysical examination
discloses bibasilar crackles a regular cardiac rhythm without gallops no enlarged abdominal
organs and 2E peripheral edema.
(aboratory studies)
0lood urea nitrogen "00 mg$d(
1erum creatinine "2.0 mg$d(
1erum sodium "*% me2$(
1erum potassium %.2 me2$(
1erum chloride 30 me2$(
1erum bicarbonate 2< me2$(
Arterial blood gas studies 7patient breathing room air8)
p! &.<0
/co2 *& mm !g
0icarbonate 22 me2$(
5hich of the following best describes this patient's acid-base status6
7A8 ;o acid-base abnormality
708 Metabolic acidosis and respiratory alkalosis
798 Metabolic acidosis and metabolic alkalosis
7:8 Despiratory acidosis and respiratory alkalosis
Nephrology - Item 45
A ##-year-old male machinist is hospitaliBed for evaluation and management of aBotemia.
/rior to admission he had seen his primary care physician because of chronic sinusitis and
headache. :uring his evaluation his serum creatinine level was *., mg$d(. =ight months ago
his serum creatinine level was ".& mg$d(. >he patient denied a history of fever chills weight
loss other systemic symptoms or exposure to any nephrotoxic drugs or contrast agents. !e
has a "%-year history of hypertension. Medications include verapamil 2<0 mg daily and
aspirin one tablet daily.
On physical examination on admission his weight is 32 kg 7"3" lb8. >emperature is normal.
!is blood pressure is "<3$,2 mm !g supine and "23$3< mm !g standing. A detailed
examination is normal
(aboratory studies 7on admission8)
!emoglobin "".* g$d(
(eukocyte count ;ormal
/latelet count ;ormal
0lood urea nitrogen #3 mg$d(
1erum creatinine *., mg$d(
1erum sodium "*# me2$(
1erum potassium *., me2$(
1erum chloride "00 me2$(
1erum bicarbonate 22 me2$(
1erum calcium "<." mg$d(
1erum phosphorus <.2 mg$d(
1erum albumin <.2 g$d(
4rinalysis 1pecific gravity ".00, p! #.0 "E protein no heme no casts
A chest radiograph shows an interstitial infiltrate and a left paratracheal lymph node.
>reatment is begun with aggressive intravenous saline hydration followed by intravenous
furosemide and subcutaneous calcitonin. >he following morning his serum creatinine level is
*.2 mg$d( and his serum calcium level is "2.# mg$d(. Additional laboratory test results show
serum parathyroid hormone of "3 pg$m( 7normal) 3 to #% pg$m(8 "2%-dihydroxyvitamin : of
3, pg$m( 7normal) "0 to %0 pg$m(8 and normal serum and urine immunoelectrophoresis
without a monoclonal M-spike.
5hich of the following is most likely causing this patient's impaired renal function6
7A8 !yperparathyroidism
708 1arcoidosis
798 Multiple myeloma kidney
7:8 Metastatic lung cancer
7=8 5egener's granulomatosis
Nephrology - Item 46
>reatment is continued for the patient in the preceding 2uestion. 0y the third hospital day his
serum creatinine level has fallen to 2." mg$d( and his serum calcium level has decreased to
"0.# mg$d(. A paratracheal lymph node biopsy specimen shows six foci of histiocytes and
giant cells. 1tains for acid-fast bacilli and fungi are negative. 1erum angiotensin-converting
enByme level is "2& 4$( 7normal) ", to ,% 4$(8.
5hich of the following is the most appropriate therapy for this patient6
7A8 /rednisone
708 /rednisone and melphalan
798 9yclophosphamide hydroxydaunomycin 7doxorubicin8 Oncovin 7vincristine8 and
prednisone 79!O/8
7:8 Fntravenous immune globulin
Nephrology - Item 47
A %#-year-old female smoker with poorly controlled hypercholesterolemia and the recent onset
of crescendo angina was hospitaliBed for coronary artery bypass surgery after cardiac
catheteriBation revealed triple vessel disease. >here was no history of diabetes mellitus
hypertension or renal disease. !er admission serum creatinine level was 0., mg$d(. !er
operative course was marked by severe hypotension. /ostoperatively she continued to have
hypotension without signs of congestive heart failure or fluid overload and dopamine and
volume resuscitation were started. >he following day blood cultures grew /seudomonas
aeruginosa and gentamicin and ticarcillin were begun. !er serum creatinine level was ".*
mg$d(.
On postoperative day 2 vasopressor therapy is continued. !er temperature is *&.0 ?9 7,3.#
?@8. !er pulse rate is "*0$min respiratory rate is "3$min and blood pressure is ""0$#0 mm
!g. 0ilateral crackles and an 1* gallop are heard. Abdominal examination is normal. 1acral
and *E peripheral edema are present. ;eurologic examination shows that she is responsive
but lethargic. there are no focal neurologic findings.
(aboratory studies)
!emoglobin "0.0 g$d(
!ematocrit *"+
(eukocyte count "3000$m(
/latelet count <%#000$m(
0lood urea nitrogen *# mg$d(
1erum creatinine 2.* mg$d(
1erum sodium "** me2$(
1erum potassium %., me2$(
1erum chloride "0" me2$(
1erum bicarbonate "& me2$(
4rinalysis 1pecific gravity ".003. E" protein E" blood no glucose. 2-* erythrocytes *
leukocytes$hpf many granular casts
4rine output the previous 2< hours was "30 m(. 4rine output does not increase appreciably
after administration of <0 mg of furosemide and % mg of bumetanide over the next < hours.
5hich of the following is most appropriate at this time6
7A8 /erform renal biopsy
708 0egin dialysis
798 Obtain renal ultrasonography
7:8 :iscontinue aminoglycoside therapy and observe
Nephrology - Item 48
A 23-year-old man is brought to the emergency department because of a <-day history of
severe watery diarrhea vomiting and poor oral fluid intake. On physical examination his
temperature is *3.0 ?9 7"00.< ?@8. !is pulse rate is ,0$min seated and "2<$min standing and
his blood pressure is "20$&% mm !g seated and 30$%0 mm !g standing. A stool specimen is
"E positive for occult blood. =xamination is otherwise normal.
(aboratory studies)
0lood urea nitrogen #< mg$d(
1erum creatinine ".< mg$d(
1erum sodium "<2 me2$(
1erum potassium *.< me2$(
1erum chloride "0# me2$(
1erum bicarbonate 2" me2$(
1erum calcium ,.0 mg$d(
4rinalysis 1pecific gravity ".02, p! %.%. 2-* leukocytes$hpf. no casts
>he elevated blood urea nitrogen)serum creatinine ratio is most likely due to which of the
following mechanisms6
7A8 =nhanced renal sodium water and urea reabsorption as a result of volume-mediated
stimulation of anti-diuretic hormone and renin-angiotensin-aldosterone
708 Fncreased urea production as a result of gastrointestinal bleeding
798 1hift of urea from the intracellular space to the extracellular space
7:8 @ever
Nephrology - Item 49
A 22-year-old white woman is evaluated because of arthralgias low-grade fever malaise and
Daynaud's phenomenon. On physical examination her temperature is *3.2 ?9 7"00.3 ?@8
pulse rate is ,<$min and blood pressure is "<%$,2 mm !g. A malar erythematous facial blush
and livedo reticularis are present. 9ardiac examination discloses a grade 2$# systolic eGection
murmur. >he metacarpophalan-geal and proximal interphalangeal Goints are swollen and there
is 2E bilateral ankle edema.
(aboratory studies)
!ematocrit 2<+
(eukocyte count 2#00$-(
/latelet count "02000$-(
0lood urea nitrogen *# mg$d(
1erum creatinine 2.< mg$d(
1erum total complement Deduced
Antinuclear antibody /ositive 7")#<08
Anti-:;A antibody /ositive 7")3<08
4rinalysis <E protein *E heme. 3-"% erythrocytes$hpf erythrocyte casts
2<-!our urinary protein 3 g$2< h
Ff a renal biopsy is performed which of the following is most likely to be found6
7A8 Membranous lupus nephropathy
708 Mesangial lupus nephritis
798 @ocal proliferative lupus nephritis
7:8 :iffuse proliferative lupus nephritis
Nephrology - Item 50
A 22-year-old white woman is evaluated because of arthralgias low-grade fever malaise and
Daynaud's phenomenon. On physical examination her temperature is *3.2 ?9 7"00.3 ?@8
pulse rate is ,<$min and blood pressure is "<%$,2 mm !g. A malar erythematous facial blush
and livedo reticularis are present. 9ardiac examination discloses a grade 2$# systolic eGection
murmur. >he metacarpophalan-geal and proximal interphalangeal Goints are swollen and there
is 2E bilateral ankle edema.
(aboratory studies)
!ematocrit 2<+
(eukocyte count 2#00$-(
/latelet count "02000$-(
0lood urea nitrogen *# mg$d(
1erum creatinine 2.< mg$d(
1erum total complement Deduced
Antinuclear antibody /ositive 7")#<08
Anti-:;A antibody /ositive 7")3<08
4rinalysis <E protein *E heme. 3-"% erythrocytes$hpf erythrocyte casts
2<-!our urinary protein 3 g$2< h
5hich of the following is the most appropriate initial therapy for the patient6
7A8 Monthly pulse intravenous methylprednisolone
708 Oral glucocorticoids at high doses for at least # months
798 Monthly pulse intravenous cyclophosphamide plus oral glucocorticoids
7:8 Oral cyclosporine plus oral glucocorticoids
Nephrology - Item 51
A patient has the laboratory test results shown below)
1erum sodium "*3 me2$(
1erum potassium <., me2$(
1erum chloride "00 me2$(
1erum bicarbonate "% me2$(
Arterial blood gas studies 7patient breathing room air8)
p! &.<3
/co2 2" mm !g
0icarbonate "% me2$(
5hich of the following patients is most likely to have these acid-base parameters6
7A8 A 2-year-old child who has accidentally swallowed an unknown 2uantity of his mother's
aspirin
708 A 23-year-old woman with bulimia and vomiting
798 A *%-year-old man who has ingested a large amount of aspirin in a suicide attempt
7:8 A %0-year-old woman with surreptitious laxative abuse
Nephrology - Item 52
A *<-year-old white male accountant is seen for a routine follow-up visit 2 years after
undergoing cadaveric renal transplantation for end-stage renal disease secondary to
glomerulonephritis. !e states that he feels well except for occasional fatigue. !e has had
excellent allograft function on an immunosuppressive regimen of prednisone &.% mg daily.
cyclosporine "2% mg twice daily. and aBathioprine "2% mg daily.
On physical examination he looks well. 0lood pressure is "%3$33 mm !g. >he remainder of
the examination is normal.
(aboratory studies)
!emoglobin "&., g$d(
!ematocrit %#+
(eukocyte count &<%0$-(. normal differential
/latelet count 2"3000$-(
1erum creatinine ".2 mg$d(
1erum cyclosporine ""% ng$m( 7target) "00 to 22% ng$m(8
Aou note that his hematocrit has increased steadily from <*+ to %#+ over the past # to ,
months and order measurement of erythrocyte mass and plasma volume. >he erythrocyte
mass is reported to be elevated and the plasma volume is normal.
After initial phlebotomy which of the following long-term therapeutic options is most
appropriate6
7A8 9hange aBathioprine to mycophenolate
708 0egin an angiotensin-converting enByme inhibitor
798 9hange cyclosporine to tacrolimus
7:8 0egin melphalan
Nephrology - Item 53
A &2-year-old man is brought to the emergency department because of anorexia
disorientation and lassitude. !e has been in good health since a motor vehicle accident "2
years earlier which necessitated extensive abdominal surgery. >here is no history of diabetes
mellitus hypertension or renal disease. !e last saw a physician # years ago.
On physical examination his pulse rate is ""3$min and regular and blood pressure is "&0$"00
mm !g without orthostatic changes. >here is moderate neck vein distention. 0ilateral crackles
are audible and an 1* gallop is heard at the left sternal border. !is abdomen is nontender no
bruit is heard and there are no enlarged abdominal organs. /opliteal and pedal pulses are
normal. >here is no discoloration of his toes or pedal edema. >he patient is somnolent but
oriented.
(aboratory studies)
!ematocrit *2+
Mean corpuscular volume ;ormal
(eukocyte count %%00$m(
/latelet count ",,000$m(
0lood urea nitrogen ""* mg$d(
1erum creatinine 3.& mg$d(
1erum sodium "*, me2$(
1erum potassium %.# me2$(
1erum chloride ,3 me2$(
1erum bicarbonate "3 me2$(
4rinalysis 1pecific gravity ".0"". trace protein no blood glucose or ketones. normal
microscopic examination
4rinary sodium <& me2$(
Denal ultrasonography shows normal-siBed kidneys and no evidence of hydronephrosis. 4rine
output over the first "2 hours of hospitaliBation is <30 m(.
5hich of the following is the most appropriate first step in managing this patient6
7A8 Obtain 9> of the kidneys and pelvis without contrast
708 Administer volume repletion
798 Obtain emergent renal arteriography
7:8 Administer pulse intravenous glucocorticoids
7=8 =nroll the patient in chronic end-stage renal disease hemodialysis program after
obtaining informed consent
Nephrology - Item 54
A #0-year-old man has chronic obstructive pulmonary disease and congestive heart failure.
9urrent medications are digoxin theophylline loop diuretics angiotensin-converting enByme
inhibitors and spironolactone. !e is hospitaliBed because of an exacerbation of his lung
disease caused by an intercurrent infection. !is clinical condition deteriorates and
endotracheal intubation and mechanical ventilation are re2uired.
After " week he has improved but the pulmonologists are having difficulty weaning him from
the ventilator. !e is currently receiving furosemide 30 mg every 3 hours to maintain diuresis.
/hysical examination shows bilateral crackles and 2E peripheral edema.
(aboratory studies)
0lood urea nitrogen %2 mg$d(
1erum creatinine 2." mg$d(
1erum sodium "*" me2$(
1erum potassium %.% me2$(
1erum chloride 30 me2$(
1erum bicarbonate *3 me2$(
Arterial blood gas studies 7patient on the ventilator8)
p! &.<3
/co2 %0 mm !g
0icarbonate *# me2$(
5hich of the following would be most helpful in weaning this patient from the ventilator6
7A8 9ontinue aggressive diuresis with loop diuretics
708 :iscontinue all diuretics and begin normal saline "00 m($h
798 9hange furosemide to acetaBolamide
7:8 0egin continuous arteriovenous ultrafiltration
Nephrology - Item 55
A #"-year-old female teacher re2uests a second opinion regarding her recently diagnosed
Kwhite coatK hypertension. @or the past < months her blood pressure readings 7measured in
the office by both her primary care physician and his nurse8 have averaged "#%$,0 mm !g.
!owever repeated blood pressure measurements that she obtains at home average "*0$&#
mm !g. 1he re2uests your opinion about the diagnosis and her risk for cardiovascular disease.
5hich of the following statements is correct6
7A8 >he risk of hypertensive cardiovascular complications 7including left ventricular
hypertrophy8 correlates more closely with office blood pressure readings than with 2<-hour or
daytime ambulatory blood pressure readings
708 /hysicians should routinely measure the office blood pressure of their patients because
readings obtained by physicians are lower than those obtained by nurses
798 1ince Kwhite coatK hypertension may affect almost 20+ of patients with mild office
hypertension all patients with this suspected diagnosis should undergo ambulatory blood
pressure monitoring with an automated device
7:8 /atients with Kwhite coatK hypertension have a higher systemic vascular resistance and
left ventricular mass index than their normotensive counterparts and may have an increased
risk of cardiovascular disease
Nephrology - Item 56
A *#-year-old obese pregnant woman is referred for evaluation. 1he has four healthy children
following uncomplicated term pregnancies. 1he has had hypertension for % years that is
currently well controlled with ramipril. >here is also a family history of hypertension. 1he was
first seen for prenatal care by a physician 2 months after her last menstrual period. !er
pregnancy had been uneventful and she had gained 2.& kg 7# lb8. !er blood pressure was
"<0$,2 mm !g and she had trace edema.
(aboratory studies)
!ematocrit **+
0lood urea nitrogen "" mg$d(
1erum creatinine 0.& mg$d(
1erum uric acid <." mg$d(
4rinalysis >race protein
4rinary protein "2% mg$2< h
9reatinine clearance "#0 m($min
5hich of the following statements is not correct regarding this patient6
7A8 !er ramipril should be discontinued
708 1he should be started on another antihypertensive agent
798 1he should be told that she has an approximately 2%+ risk of developing preeclampsia
7:8 1he should be told that better control of blood pressure will diminish her risk of
developing preeclampsia
Nephrology - Item 57
An otherwise healthy <"-year-old man has had recurrent calcium oxalate nephrolithiasis for
the past * years. A recent 2<-hour urine collection showed the following)
9reatinine ".%# g$2< h
9alcium *30 mg$2< h
4rate &<0 mg$2< h 7normal) H &%0 mg$2< h8
Oxalate *3 mg$2< h 7normal) H <0 mg$2< h8
9itrate #<* mg$2< h 7normal) *00 to &00 mg$2< h8
1odium "0< me2$2< h
5hich of the following would be most effective in reducing his urinary calcium excretion6
7A8 :ietary calcium restriction
708 9ranberry Guice
798 !ydrochlorothiaBide
7:8 @urosemide
7=8 !igh fluid intake
Nephrology - Item 58
5hich of the following genetic renal disorders is usually associated with renal failure6
7A8 Medullary sponge kidney
708 ;ephrogenic diabetes insipidus
798 >hin basement membrane disease
7:8 ;ephronophthisis and medullary cystic disease
Nephrology - Item 59
A 23-year-old woman with cystic fibrosis was hospitaliBed 2 weeks ago because of fever
cough purulent yellow-green sputum and a new right lower lobe infiltrate. 1he has been
treated with tobramycin and piperacillin intravenously since admission. !er renal function
which was normal on admission has deteriorated. !er blood urea nitrogen level has increased
from "< mg$d( to *2 mg$d( and her serum creatinine concentration has risen from 0.# mg$d(
to *.2 mg$d(. >here has been no decrease in urine output during this period. Additional
medications have included ibuprofen for chest discomfort.
On physical examination her temperature is *3.* ?9 7"0".0 ?@8. !er pulse rate is 33$min
respiratory rate is "<$min and unlabored and blood pressure is "*0$&2 mm !g. >here is a
faint blanching maculopapular rash on her trunk and upper extremities. =xamination of her
chest shows only scattered rhonchi. >he remainder of the examination is normal.
(aboratory studies)
!ematocrit *3+
(eukocyte count &%00$-(
/latelet count *2%000$-(
0lood urea nitrogen *2 mg$d(
1erum creatinine *.2 mg$d(
1erum electrolytes ;ormal
4rinalysis >race protein <E heme. many leukocytes and erythrocytes$hpf no
erythrocyte casts
9hest radiograph confirms that the pulmonary infiltrate is resolving.
5hich of the following is the most appropriate diagnostic study at this time6
7A8 Fntravenous pyelography
708 5right or !ansel stain of the urine sediment
798 :etermination of serum tobramycin level
7:8 2<-!our urine for calcium and oxalate
Nephrology - Item 60
A 2*-year-old man is evaluated because of increasing dyspnea. !e has a history of
intravenous drug use and known !FI infection. 9hest radiograph shows bilateral infiltrates.
>he patient is hospitaliBed. !is blood pressure on admission is "20$&# mm !g.
(aboratory studies on admission)
0lood urea nitrogen 20 mg$d(
1erum creatinine ".* mg$d(
1erum sodium "*" me2$(
1erum potassium <.3 me2$(
1erum chloride ,% me2$(
1erum bicarbonate 22 me2$(
@urther evaluation leads to a diagnosis of /neumocystis carinii pneumonia and parenteral
trimethoprimsulfamethoxaBole is begun.
(aboratory studies one week after admission)
0lood urea nitrogen 2% mg$d(
1erum creatinine ".3 mg$d(
1erum sodium "*0 me2$(
1erum potassium #.2 me2$(
1erum chloride ,* me2$(
1erum bicarbonate 2< me2$(
5hich of the following is the most likely cause of the increased potassium concentration in this
patient6
7A8 (ung tissue destruction due to /. carinii infection
708 :ecreased renal potassium excretion due to deterior-ating renal function
798 0lockade of distal renal tubule potassium excretion due to trimethoprim
7:8 Fnterstitial nephritis due to sulfamethoxaBole
7=8 Addison's disease due to adrenal !FI infection
Nephrology - Item 61
A <3-year-old white man is evaluated because of cough arthralgias malaise and weight loss.
!e had a normal annual physical examination and renal function studies 2 months ago.
On physical examination today his temperature is *&.* ?9 7,,.2 ?@8. !is pulse rate is ,2$min
and regular respiratory rate is 2<$min and blood pressure is "#0$,< mm !g. 9ardiac
examination is normal. =xamination of the chest discloses crackles and rhonchi at the left
base. =xamination of the abdomen and extremities is normal.
(aboratory studies)
!ematocrit *<+
(eukocyte count "2000$-(
/latelet count <2%000$-(
/lasma glucose ;ormal
0lood urea nitrogen 2% mg$d(
1erum creatinine ".3 mg$d(
1erum electrolytes ;ormal
4rinalysis
*E protein <E heme. many erythrocytes$hpf many dysmorphic erythrocytes
2<-!our urinary protein 2.& g$2< h
A chest radiograph shows a haBy left lower lobe infiltrate. An electrocardiogram is normal.
Denal ultrasonography shows normal kidney siBe and texture.
5hich of the following diagnostic studies should be done next6
7A8 :eterminations of serum !FI antibodies and !FI viral load
708 Antineutrophil cytoplasmic antibody 7A;9A8 and anti-glomerular basement membrane
7anti-C0M8 antibody assays
798 1erum and urine protein electrophoresis
7:8 1erologic tests for hepatitis 0 and 9
Nephrology - Item 62
A renal biopsy is performed on the patient in the preceding 2uestion. >he biopsy specimen
shows focal and segmental necrotiBing glomerulonephritis with crescent formation.
Fmmunofluorescence shows no immune staining.
5hich of the following is the most appropriate initial therapy6
7A8 9yclophosphamide and glucocorticoids
708 9yclophosphamide glucocorticoids and plasmapheresis
798 9yclosporine and glucocorticoids
7:8 9yclosporine glucocorticoids and plasmapheresis
Nephrology - Item 63
A %#-year-old white woman re2uests a second opinion regarding her primary care physician's
diagnosis of Kmicro-protein in my urineK " month ago. 1he has a #-year history of type 2
diabetes mellitus and hypertension. Medical history is otherwise unremarkable. Medications
include glipiBide % mg each morning. sustained-release nifedipine *0 mg daily. and daily
aspirin. 1he follows an American :iabetes Association diet.
On physical examination the patient appears well. 1he weighs <& kg 7"0* lb8. 0lood pressure
is "<3$,2 mm !g seated and standing. @unduscopic examination shows background diabetic
retinopathy without hemorrhages exudates or papilledema. >here is no peripheral edema.
>he remainder of the examination is normal.
(aboratory studies)
9omplete blood count ;ormal
!emoglobin A"9 #.*+ 7normal) <.0+ to #.0+8
/lasma glucose 7fasting8 "2" mg$d(
0lood urea nitrogen ;ormal
1erum creatinine ;ormal
1erum electrolytes ;ormal
1erum thyroid-stimulating hormone ;ormal
4rinalysis Doutine urinalysis normal. microalbumin assay of the albumin)creatinine ratio
%0 mg$g
>he patient re2uests your opinion regarding her diagnosis of microalbuminuria and its
implications regarding her therapy and risks for renal disease.
5hich of the following statements is correct6
7A8 >he presence of microalbuminuria in a patient with type " or type 2 diabetes mellitus is
usually indicative of early diabetic nephropathy
708 Almost all patients with type 2 diabetes mellitus who have microalbuminuria develop
overt nephropathy over a "0-year-period
798 @or patients with type 2 diabetes mellitus and essential hypertension the risk of
cardiovascular disease is not influenced by the presence of microalbuminuria
7:8 Optimal therapy for diabetic patients with microalbuminuria who are taking nifedipine is
to increase the nifedipine dose to optimiBe control of blood pressure
Nephrology - Item 64
A 23-year-old man is found to have microscopic hematuria on an insurance physical
examination. !is physical exam-ination in your office is normal. 9omplete blood count plasma
glucose blood urea nitrogen serum creatinine and serum electrolytes are normal. 4rinalysis
shows no protein many erythrocytes$hpf no erythrocyte casts and no dysmorphic
erythrocytes. 4rine culture shows no growth and an intravenous pyelogram is reported as
showing medullary sponge kidney but no stones or other abnormalities.
Appropriate counseling of this patient includes which of the following6
7A8 Advise him that this disorder is likely to progress to chronic renal failure over "0 to 20
years
708 Advise him that this is a benign finding and that although it may be a risk factor for
nephrolithiasis it never leads to progressive renal failure
798 Advise him to have his children undergo genetic testing for this disorder so that they can
be treated appropriately at an early age
7:8 Advise him that the course of this disorder can be modified by treatment with an
angiotensin-converting enByme inhibitor
Nephrology - Item 65
A *"-year-old woman is evaluated 2 days after she passed a kidney stone following * days of
left-sided renal colic. 1he has a "0-year history of recurrent episodes of calcium phosphate
nephrolithiasis. At age "& she developed inflammatory bowel disease. @or the past 2 years
she has had symptoms of dry eyes dry mouth and Daynaud's phenomenon.
(aboratory studies)
0lood urea nitrogen "% mg$d(
1erum creatinine ".2 mg$d(
1erum sodium "<2 me2$(
1erum potassium 2., me2$(
1erum chloride ""2 me2$(
1erum bicarbonate 20 me2$(
4rinalysis 1pecific gravity ".0"% p! #.% 2E heme no protein *-"0 erythrocytes$hpf
A plain radiograph of the abdomen and pelvis shows multiple "- to 2-mm calcifications
overlying both renal shadows.
5hich of the following is the most likely cause of the renal stone disease in this patient6
7A8 Fdiopathic hypercalciuria
708 9ystinuria
798 Denal tubular acidosis
7:8 1truvite 7infection8 stone disease
7=8 Medullary sponge kidney
Nephrology - Item 66
A #%-year-old man is seen for a routine annual evaluation. !e states that he has been feeling
fatigued and exhausted. !e is normotensive and his physical examination is normal.
(aboratory studies)
!ematocrit 2%+
(eukocyte count %#00$-(
/latelet count *<0000$-(
/lasma glucose &3 mg$d(
0lood urea nitrogen 23 mg$d(
1erum creatinine 2." mg$d(
1erum sodium "*% me2$(
1erum potassium %.0 me2$(
1erum chloride "0% me2$(
1erum bicarbonate 23 me2$(
1erum calcium "0., mg$d(
1erum phosphorus <., mg$d(
4rinalysis >race protein.
normal sediment
9reatinine clearance <3 m($min
2<-!our urinary protein 2.3 g$2< h
5hich of the following is the most appropriate diagnostic test at this time6
7A8 Fntravenous pyelography
708 4rine immunoelectrophoresis
798 1erum angiotensin-converting enByme determination
7:8 1erum complement determination and antinuclear antibody assay
Nephrology - Item 67
A *0-year-old man has a history of a seiBure disorder follow-ing head trauma % years ago.
>reatment with phenytoin has controlled his seiBures ade2uately. !e has a grand mal seiBure
while KpartyingK late one evening and is brought to the emergency department by his friends.
>he friends state that they had all been drinking alcohol but deny use of any illicit drugs or
stimulants.
On physical examination the patient is a well-developed muscular young man who is very
lethargic and confused. !e is afebrile. !is pulse rate is "00$min and regular respiratory rate is
"3$min and deep and blood pressure is "*0$,% mm !g. =xamination is unremarkable except
for the lethargy and confusion.
(aboratory studies)
9omplete blood count ;ormal
0lood urea nitrogen "2 mg$d(
1erum creatinine ".0 mg$d(
1erum sodium "<0 me2$(
1erum potassium <.3 me2$(
1erum chloride "00 me2$(
1erum bicarbonate "2 me2$(
1erum osmolality 7measured8 *"0 mosm$(
Arterial blood gas studies 7patient breathing room air8)
p! &.2%
/co2 23 mm !g
0icarbonate 7calculated8 "2 me2$(
0lood ethanol "00 mg$d(
4rinalysis ;ormal
5hich of the following is the most appropriate initial treatment for the metabolic acidosis and
osmotic gap in this patient6
7A8 Observation and repeat arterial blood gas studies in 2 hours
708 1odium bicarbonate 2 ampules 7"00 me28 by intravenous push
798 One liter of %+ dextrose in water plus bicarbonate * ampules 7"%0 me28 infused over *
hours
7:8 !emodialysis
7=8 @omepiBole
Nephrology - Item 68
A 2*-year-old female graduate student re2uests a second opinion because of
Kmicroalbuminuria and normal blood pressure.K >he patient has had type " diabetes mellitus
for , years managed with diet exercise and split-dose insulin therapy. 1he states that she
carefully adheres to her diabetes regimen. !er primary care physician noted microalbuminuria
during routine screening last month. 1he denies other target organ damage due to diabetes.
0lood pressure determinations both in the office and at home average "*0$3% mm !g.
On physical examination she appears well. !er weight is #2 kg 7"*# lb8. 0lood pressure is
"23$32 mm !g seated and standing. =xamination is normal.
>he following laboratory studies are normal) complete blood count plasma glucose blood urea
nitrogen serum creatinine and serum electrolytes. !emoglobin A"9 is %.3+ 7normal) <.0+ to
#.0+8. Doutine urinalysis is normal.
>he patient's primary care physician wants to begin anti-hypertensive therapy for her recently
discovered microal-buminuria. 1he is very concerned about the possibility of developing renal
disease but also is concerned about the advisability of taking antihypertensive drugs since
her blood pressure is normal.
5hich of the following statements is correct6
7A8 /rogression to overt nephropathy occurs in greater than &0+ of patients with type "
diabetes mellitus and microalbuminuria especially if microalbuminuria develops more than "%
years following the diagnosis of diabetes mellitus
708 >herapy with an angiotensin-converting enByme 7A9=8 inhibitor is likely to reduce the
degree of microalbuminuria and significantly lessen the chance of progression to overt
nephropathy in patients with type " diabetes mellitus and microalbuminuria
798 >he risk of developing microalbuminuria does not correlate with glycemic control and
strict glycemic control does not reduce the risk of developing microalbuminuria
7:8 >herapy with an A9= inhibitor has been proved to lessen the chance of developing overt
nephropathy in all patients with type " diabetes mellitus even patients who do not have
hypertension or microalbuminuria
Nephrology - Item 69
A *3-year-old woman with biopsy-proven focal segmental glomerulosclerosis has a serum
creatinine level of 2.0 mg$d( a 2<-hour creatinine clearance of <3 m($min and a 2<-hour
urinary protein excretion of *.& g. !er glomerular filtration rate 7C@D8 using iothalamate as a
marker 7true C@D8 is *2 m($min.
5hich of the following mechanisms explains why the creatinine clearance overestimates the
true C@D in this patient6
7A8 Fncreased tubular secretion of creatinine
708 :ecreased reabsorption of filtered creatinine
798 Fncreased release of creatinine from muscle
7:8 :ecreased tubular secretion of creatinine
Nephrology - Item 70
A %%-year-old man is evaluated because of a "0-day history of a severe sore throat. One year
ago he had a soft tissue sarcoma of his right thumb leading to surgical amputation radiation
therapy to his right axilla and chemotherapy with carboplatin and ifosfamide. At that time
extensive evaluation revealed no metastases and normal renal function.
On physical examination his temperature is *&.2 ?9 7,,.0 ?@8. !is pulse rate is 33$min and
regular respiratory rate is "<$min and blood pressure is "*0$3# mm !g. >he right thumb is
absent. =xamination is otherwise normal.
(aboratory studies)
9omplete blood count ;ormal
/lasma glucose ;ormal
0lood urea nitrogen <2 mg$d(
1erum creatinine *.3 mg$d(
1erum electrolytes ;ormal
4rinalysis ;o protein. 2-% leukocytes 2-% erythrocytes$hpf
Denal ultrasonography shows no hydronephrosis.
5hich of the following is the most likely diagnosis6
7A8 /oststreptococcal glomerulonephritis
708 FgA nephropathy
798 Dadiation nephritis
7:8 Ffosfamide interstitial nephritis
Nephrology - Item 71
A *,-year-old male computer analyst with a #-year history of autosomal dominant polycystic
kidney disease 7A:/J:8 is evaluated because of intermittent left flank pain. !e does not have
fever chills dysuria or a history of trauma. 1ince being diagnosed with A:/J: he has not
had urinary tract infections hematuria or renal calculi. Over the past < years his serum
creatinine level has slowly risen from ".< mg$d( to "., mg$d( 7at the time of his last visit <
weeks ago8. (isinopril "0 mg daily was started 2 years ago for management of stage "
hypertension 7blood pressure "%3$,2 mm !g8. !e has not taken nonsteroidal anti-
inflammatory drugs recently and has not been exposed to contrast agents or any other
nephrotoxins.
On physical examination his blood pressure is "*3$33 mm !g seated and standing.
=xamination of the head and neck and cardiopulmonary examination are normal. /olycystic
kidneys are easily palpated bilaterally and there is mild tenderness in the left flank.
;euromuscular examination is normal. >here is no peripheral edema.
(aboratory studies)
9omplete blood count ;ormal
0lood urea nitrogen <* mg$d(
1erum creatinine <.2 mg$d(
1erum sodium "*3 me2$(
1erum potassium %.* me2$(
1erum chloride "00 me2$(
1erum bicarbonate 22 me2$(
1erum calcium ,.2 mg$d(
1erum phosphorus <.2 mg$d(
1erum uric acid 3.2 mg$d(
4rinalysis >race protein 2E heme no glucose. 0-2 leukocytes "0-"% erythrocytes$hpf
no casts
A plain radiograph of the abdomen and pelvis is normal. >here are no calcifications or
radiopa2ue densities over the left renal bed ureteral tract or bladder. 9> of the abdomen
without contrast shows "%-cm kidneys bilaterally with innumerable cysts but no hemorrhage.
>here is a %-mm density at the left ureteropelvic Gunction with some hydronephrosis of the
collecting system.
5hich of the following is the most likely explanation for this patient's findings6
7A8 ;atural progression of A:/J:
708 9omplication of angiotensin-converting enByme inhibitor therapy
798 Denal calculus
7:8 !emorrhage or infection of a cyst
Nephrology - Item 72
A #0-year-old man is evaluated because of "-month history of headache and palpitations. >he
patient has progressive renal insufficiency secondary to chronic glomerulonephritis. !e also
has had chronic obstructive pulmonary disease for "0 years treated with inhaled
bronchodilators and glucocorticoids. >here is no history of hypertension. Medications include
prednisone "0 mg daily. ferrous sulfate *2% mg daily. albuterol 2 puffs < times daily. and
ipratropium bromide 2 puffs * times daily. !e has also received erythropoietin "0000 units
subcutaneously once a week for the past 2 months for the anemia associated with his renal
insufficiency. !is blood pressure 2 months ago was "*0$30 mm !g hematocrit was 2&+ and
serum creatinine was *.% mg$d(.
On physical examination the patient appears alert and comfortable. !e weighs ,& kg 72"* lb8.
>emperature is *#.# ?9 7,3.0 ?@8. !is pulse rate is ,0$min and blood pressure is "#0$""0 mm
!g. >here is no neck vein distention. >he lung fields are clear and cardiac examination is
normal. !is abdomen is obese without palpable tenderness or enlarged organs. >here is trace
pedal edema bilaterally.
(aboratory studies)
!emoglobin "2.0 g$d(
!ematocrit *#+
(eukocyte count #%00$-(
/latelet count 2#0000$-(
0lood urea nitrogen %& mg$d(
1erum creatinine *.& mg$d(
1erum sodium "*% me2$(
1erum potassium *., me2$(
1erum chloride "00 me2$(
1erum bicarbonate 20 me2$(
5hich of the following best explains this patient's new-onset hypertension6
7A8 =ssential hypertension
708 5orsening renal failure with fluid retention
798 9omplication of erythropoietin administration
7:8 Fatrogenic 9ushing's syndrome
Nephrology - Item 73
A <2-year-old woman with known sickle cell disease and longstanding nocturia and polyuria is
evaluated because of new-onset painless gross hematuria. On physical examination her
temperature is *&." ?9 7,3.3 ?@8. !er pulse rate is ""0$min and regular respiratory rate is
"<$min and blood pressure is "*%$3# mm !g. 9ardiac examination shows a grade 2$# systolic
eGection murmur. >he remainder of the physical examination is normal.
(aboratory studies)
!ematocrit 2&+
(eukocyte count %000$-(
/latelet count *20000$-(
/lasma glucose ;ormal
0lood urea nitrogen ;ormal
1erum creatinine ;ormal
1erum electrolytes ;ormal
4rinalysis Many erythrocytes$hpf no erythrocyte casts
4rine culture ;o growth
5hich of the following diagnostic studies is most appro-priate at this time6
7A8 4ltrasonography of the kidneys
708 9> of the kidneys
798 9> of the kidneys and cystoscopy
7:8 Denal angiography
Nephrology - Item 74
A *,-year-old man is brought to the emergency department because of a "2-hour history of
abdominal pain and nausea. !e has chronic focal and segmental glomerulo-sclerosis with a
baseline serum creatinine concentration of 2." mg$d( and a urinary protein excretion of 2 g$2<
h. A serum creatinine determination in the emergency department is 2.0 mg$d( and 9> of the
abdomen with contrast is normal. >hirty hours later his serum creatinine is *.2 mg$d( and
urine output has fallen to 20 m($h. 0ecause of the acute renal failure you suspect a diagnosis
of contrast-induced acute tubular necrosis.
5hich of the following urinalysis findings is most compatible with your diagnosis6
7A8 1pecific Cravity ".0"2
/rotein 2E
0lood 2E
Microscopic =xamination 20-*0 erythrocytes "%-20 leukocytes$hpf. !ansel's
stain positive for eosinophils 7H %+ leukocytes8
708
1pecific Cravity ".0"0
/rotein 2E
0lood ;egative
Microscopic =xamination "-* leukocytes %-"0 renal tubular cells$hpf. many
pigmented granular casts occasional renal tubular cell casts. !ansel's stain negative
798
1pecific Cravity ".0"2
/rotein 2E
0lood 2E
Microscopic =xamination %-"0 erythrocytes 2%-%0 leukocytes$hpf. many
bacteria occasional finely granular casts. !ansel's stain negative
7:8
1pecific Cravity ".020
/rotein 2E
0lood 2E to *E
Microscopic =xamination "0-20 erythrocytes 2-< leukocytes$hpf. "-*
erythrocyte casts$hpf. !ansel's stain negative
Nephrology - Item 75
A %%-year-old white woman with chronic low back pain and chronic polyuria and nocturia is
found to have renal dysfunction during a routine annual evaluation. 1he has no significant
medical history and does not use any medications. /hysical examination is normal.
(aboratory studies)
9omplete blood count ;ormal
/lasma glucose ;ormal
0lood urea nitrogen 23 mg$d(
1erum creatinine 2.* mg$d(
1erum electrolytes ;ormal
4rinalysis ;o protein. no erythrocytes %-"0 leukocytes$hpf no casts
4rine culture ;o growth
Denal ultrasonography suggests normal-siBed kidneys without hydronephrosis but with
papillary necrosis.
5hich of the following should be done next6
7A8 1creen the patient for hemoglobin 1A trait
708 /erform a glucose tolerance test
798 Obtain a urine culture for Mycobacterium tuberculosis
7:8 Ask the patient again about use of prescribed drugs and over-the-counter medications
Nephrology - Item 76
A *0-year-old woman has a long history of 9rohn's disease. 1he has re2uired multiple
abdominal procedures and has an ileostomy. 1he usually controls her ostomy output with
loperamide. @or the past week she has noted increased ostomy output weakness and
paresthesias. 1he comes to the emergency department and is subse2uently hospitaliBed.
/hysical examination on admission discloses that her blood pressure is ""%$&% mm !g supine
and "0%$#% mm !g seated. !er skin is normal. 9ardiopulmonary examination is normal. !er
abdomen is not tender. 1he has multiple surgical scars and a functioning ileostomy. >here is
"E peripheral edema.
(aboratory studies on admission)
0lood urea nitrogen *0 mg$d(
1erum creatinine 2.0 mg$d(
1erum sodium "2, me2$(
1erum potassium 2., me2$(
1erum chloride 3% me2$(
1erum bicarbonate "3 me2$(
1erum calcium %.% mg$d(
1erum phosphorus ".* mg$d(
1erum albumin *.2 g$d(
Fntravenous fluids consisting of alternating bags of isotonic saline plus <0 me2$( of potassium
chloride and half-normal isotonic saline plus <0 me2$( of potassium chloride and %0 me2$( of
sodium bicarbonate are begun at "%0 m($h. After 2< hours the orthostatic changes are
reversed. !er blood urea nitrogen level has fallen to 20 mg$d( and her serum creatinine
concentration has decreased to ".% mg$d(. 1erum bicarbonate concentration is 22 me2$(.
!owever her serum potassium level is still 2., me2$( and her serum calcium level is %.*
mg$d(.
5hich of the following is the most likely reason for the persistent hypokalemia6
7A8 >he patient is likely magnesium depleted which prevents potassium repletion
708 >he persistent hypocalcemia makes correction of hypokalemia more difficult
798 >he administered sodium bicarbonate is shifting potassium into cells
7:8 >he hypophosphatemia has exacerbated the hypokalemia
Nephrology - Item 77
A #*-year-old woman with diabetic nephropathy comes to your office for a routine visit after
returning from a 9aribbean cruise. 1he has a 20-year history of hypertension and type 2
diabetes mellitus. Medications include lisinopril "0 mg daily and a calcium supplement. >he
patient is concerned about her diabetes mellitus and kidney disease because of Kdietary
indiscretionsK on the cruise ship.
On physical examination her pulse rate is ,0$min and blood pressure is "%0$,2 mm !g.
@unduscopic examination shows hypertensive diabetic retinopathy. 9ardiopulmonary and
abdominal examinations are normal. >here are no neuromuscular deficits but 2E bilateral
pedal edema is noted.
(aboratory studies)
!ematocrit 23+
0lood urea nitrogen %< mg$d(
1erum creatinine <., mg$d( 7stable value8
1erum calcium 3.3 mg$d(
1erum phosphorus &.% mg$d(
1erum uric acid "0., mg$d(
1erum albumin 2., g$d(
1erum parathyroid hormone 2%3 pg$m( 7normal) H %0 pg$m(8
1erum protein electrophoresis ;ormal
4rine protein electrophoresis ;ormal
Fn addition to recommending a low-phosphorus diet which of the following is the most
appropriate initial therapy for this patient's hyperphosphatemia6
7A8 9alcium-containing phosphate binders with meals
708 "2%-:ihydroxyvitamin : 7Docaltrol8 each morning
798 Aluminum hydroxide with meals
7:8 Magnesium hydroxide with meals
Nephrology - Item 78
An "3-year-old man becomes despondent after an argument with his girlfriend. !e swallows a
KpoisonK and * hours later comes to the emergency department. !e states that he threw the
bottle into a sewer and it cannot be recovered. !e is not aware of the specific chemical that
he ingested but knows only that the bottle had a skull and crossbones on the label.
>he patient is disheveled and anxious. /hysical examination is otherwise normal.
(aboratory studies)
0lood urea nitrogen *0 mg$d(
1erum creatinine ".3 mg$d(
1erum sodium "<0 me2$(
1erum potassium %.* me2$(
1erum chloride "00 me2$(
1erum bicarbonate "0 me2$(
Arterial blood gas studies 7patient breathing room air8)
p!&."#
/co2 2* mm !g
/o2 "00 mm !g
0icarbonate 3 me2$(
4rinalysis Many erythrocytes$hpf many rectangular crystals
5hich of the following poisons did this patient most likely ingest6
7A8 =thylene glycol
708 Methanol
798 Fsopropyl alcohol
7:8 9yanide
7=8 9alcium oxalate
Nephrology - Item 79
A %0-year-old woman is evaluated because of KspellsK characteriBed by throbbing headache
hypertension chest discomfort with palpitations nausea diBBiness and sweating. >hese
episodes have occurred periodically throughout her life but both the fre2uency and the
intensity have increased recently. 1he also reports a history of high blood pressure and her
former primary care physician had tried multiple combinations of medications with only
moderate success. 9urrent medications include an estradiol patch 0.2% mg daily. felodipine
"0 mg daily. labetalol *00 mg twice daily. hydrochlorothiaBide 2% mg daily. and aspirin 3"
mg daily. On physical examination the patient is thin and anxious but is in no acute distress.
!er weight is %<.2 kg 7"", lb8. !er pulse rate is ,0$min respiratory rate is "3$min and blood
pressure is "3,$,, mm !g supine and "%<$3< mm !g standing. >he remainder of the physical
examination is normal.
(aboratory studies)
9omplete blood count ;ormal
/lasma glucose 7fasting8 "2% mg$d(
0lood urea nitrogen "< mg$d(
1erum creatinine 0., mg$d(
1erum sodium "<2 me2$(
1erum potassium <.* me2$(
1erum chloride "00 me2$(
1erum bicarbonate 23 me2$(
1erum thyroid function studies ;ormal
/lasma norepinephrine ""%# pg$m( 7normal) "00 to <%0 pg$m(8
4rinary norepinephrine "2&0 nmol$2< h 7normal) %,0 to 3%% nmol$2< h8
4rinary epinephrine 2&* nmol$2< h 7normal) H 2&% nmol$2< h8
4rinary vanillylmandelic acid *2 -mol$2< h 7normal) H *% -mol$2< h8
4rinary creatinine "% mg$kg$2< h
5hich of the following statements is correct regarding this patient6
7A8 >he elevated plasma and urinary catecholamine levels are diagnostic for
pheochromocytoma and the patient should undergo laparoscopic adrenalectomy
708 An elevated plasma glucose level is found only when a pheochromocytoma is associated
with one of the multiple endocrine neoplasia 7M=;8 syndromes
798 >his patient's antihypertensive medications may increase norepinephrine levels and the
test should be repeated after proper adGustments of medications
7:8 >he diagnosis of pheochromocytoma is best approached by bilateral adrenal vein
sampling for metanephrine
7=8 /heochromocytoma is unlikely in this age group and is not associated with orthostatic
changes in blood pressure
Nephrology - Item 80
A 2*-year-old pregnant woman was seen by her obstetrician 2 months after her last menstrual
period. >his is her first pregnancy and she has had no personal or family history of diabetes
mellitus renal disease or hypertension. On physical examination her pulse rate was 32$min
and her blood pressure was ,0$#0 mm !g. !er serum creatinine concentration was 0.< mg$d(.
Over the next 2 months she was continually troubled by vomiting. !er ability to eat was
severely limited and she only gained 0., kg 72 lb8.
At her <-month visit her pulse rate is ""0$min seated and "*0$min standing and her blood
pressure is 3*$#0 mm !g seated and #0 mm !g systolic standing. >here is no Gugular venous
distention. !er chest is clear and there is no murmur gallop or rub. !er abdomen is
nontender. /elvic examination shows a "#-week gravid uterus. 1he has no pedal edema.
;eurologic examination is normal.
(aboratory studies)
!ematocrit *2+
(eukocyte count %*00$-(
/latelet count 2,,000$-(
0lood urea nitrogen 2# mg$d(
1erum creatinine ".* mg$d(
/eripheral blood smear Microcytic erythrocytes
4rinalysis 1pecific gravity ".020. trace protein no blood ketones or glucose. normal
microscopic examination
5hich of the following is the most likely cause of this patient's renal insufficiency6
7A8 /reeclampsia
708 Iomiting
798 >hrombotic microangiopathy
7:8 4rinary tract obstruction
7=8 1ystemic lupus erythematosus
Nephrology - Item 81
A 2*-year-old pregnant woman was seen by her obstetrician 2 months after her last menstrual
period. >his is her first pregnancy and she has had no personal or family history of diabetes
mellitus renal disease or hypertension. On physical examination her pulse rate was 32$min
and her blood pressure was ,0$#0 mm !g. !er serum creatinine concentration was 0.< mg$d(.
Over the next 2 months she was continually troubled by vomiting. !er ability to eat was
severely limited and she only gained 0., kg 72 lb8.
At her <-month visit her pulse rate is ""0$min seated and "*0$min standing and her blood
pressure is 3*$#0 mm !g seated and #0 mm !g systolic standing. >here is no Gugular venous
distention. !er chest is clear and there is no murmur gallop or rub. !er abdomen is
nontender. /elvic examination shows a "#-week gravid uterus. 1he has no pedal edema.
;eurologic examination is normal.
(aboratory studies)
!ematocrit *2+
(eukocyte count %*00$-(
/latelet count 2,,000$-(
0lood urea nitrogen 2# mg$d(
1erum creatinine ".* mg$d(
/eripheral blood smear Microcytic erythrocytes
4rinalysis 1pecific gravity ".020. trace protein no blood ketones or glucose. normal
microscopic examination
5hich of the following is the most appropriate treatment for this patient6
7A8 Iolume repletion with normal saline and provision of nutrition
708 /lasmapheresis
798 >herapeutic abortion
7:8 Clucocorticoids
Nephrology - Item 82
Fn which of the following clinical situations would an increase in the serum creatinine
concentration be explained only by a reduction in the glomerular filtration rate6
7A8 4se of trimethoprim in a patient with a urinary tract infection
708 Fncreased levels of ketoacids in a patient with diabetic acidosis
798 1evere extracellular volume contraction in a patient with diarrhea
7:8 4se of cimetidine in a patient with a peptic ulcer
7=8 1eiBures in a patient with status epilepticus
Nephrology - Item 83
A <3-year-old businessman is brought to the emergency department because of substernal
chest pain and weakness. An acute myocardial infarction is diagnosed and he is transferred to
the cardiac care unit. !e has a %-year history of hypertension and has been treated with
hydrochlorothia-Bide with an excellent response. >he patient smokes one pack of cigarettes a
day and drinks about two alcoholic beverages daily.
On physical examination in the cardiac care unit his pulse rate is 30$min and regular and his
blood pressure is "2#$30 mm !g without orthostatic changes. >here is no neck vein distention
or edema. 9ardiac and abdominal examinations are normal.
(aboratory studies
0lood urea nitrogen "2 mg$d(
1erum creatinine ".0 mg$d(
1erum sodium "*& me2$(
1erum potassium *." me2$(
1erum chloride ,, me2$(
1erum bicarbonate 23 me2$(
1erum calcium &.3 mg$d(
1erum magnesium 0.& mg$d(
4rinalysis
1pecific gravity ".0"*. trace protein no ketones or glucose. normal microscopic examination
An electrocardiogram shows normal sinus rhythm 1> elevations in the inferior leads and
peaked > waves across the precordial leads.
5hich of the following is the most appropriate first step in correcting the electrolyte disorder in
this patient6
7A8 :iscontinue hydrochlorothiaBide
708 Administer magnesium sulfate intravenously
798 Administer calcium gluconate intravenously
7:8 Administer volume repletion

You might also like